You are on page 1of 75

Table of Contents

LEGAL REASONING ........................................................................................................................... 2


NLSIU ................................................................................................................................................... 2
1992................................................................................................................................................. 2
1993................................................................................................................................................. 3
1994................................................................................................................................................. 3
1995................................................................................................................................................. 4
1996................................................................................................................................................. 5
1997................................................................................................................................................. 7
1998................................................................................................................................................. 8
1999................................................................................................................................................. 8
2000................................................................................................................................................. 9
2001............................................................................................................................................... 10
2002............................................................................................................................................... 11
2003............................................................................................................................................... 12
2004............................................................................................................................................... 12
2005............................................................................................................................................... 13
2006............................................................................................................................................... 13
2007............................................................................................................................................... 14
NALSAR.............................................................................................................................................. 15
2000............................................................................................................................................... 15
2001............................................................................................................................................... 15
2002............................................................................................................................................... 16
2003............................................................................................................................................... 18
2004............................................................................................................................................... 21
2005............................................................................................................................................... 22
2006............................................................................................................................................... 24
2007............................................................................................................................................... 25
NLU-D ................................................................................................................................................ 26
2008............................................................................................................................................... 26
2009............................................................................................................................................... 29
2010............................................................................................................................................... 32

© Clat Possible. All rights reserved. Unauthorized copying, sale, distribution or circulation
of any of the contents of this work is a punishable offence under the laws of India.
www.clatpossible.com
A Team Satyam Offering
2011............................................................................................................................................... 34
2012............................................................................................................................................... 36
2013............................................................................................................................................... 39
2014............................................................................................................................................... 42
2015............................................................................................................................................... 45
2016............................................................................................................................................... 47
CLAT .................................................................................................................................................. 50
2010............................................................................................................................................... 50
2011............................................................................................................................................... 52
2012............................................................................................................................................... 54
2013............................................................................................................................................... 56
2014............................................................................................................................................... 60
2015............................................................................................................................................... 64
2016............................................................................................................................................... 66
NUJS .................................................................................................................................................. 69
2004............................................................................................................................................... 69
2007............................................................................................................................................... 71

LEGAL REASONING
NLSIU
1992
SECTION IV, PART A:
Q. no. Answer
1 The rule is when nurse is working on hospital roll and working under doctor who was an
independent contractor, so in this case both Dr. Ramana and Dr. Vikram shall be liable for the
negligence of the staff nurse.

2 Lokesh can sue Ramakrishna, because it was he who gave instructions to Kannapan as to
which work and how that work is to be done.

3 Gokul Seth is liable for the payment to be made to the supplier because Ramayya was
represented to the third party as his agent and was discharging duties on Gokul’s behalf.

4 Anil Chand shall not be liable for the expenses incurred by Kaviraj Enterprises. He did not
consent himself to be represented as a partner, nor he extended any credit to the form. He was
just a customer to them.

5. Gayaram’s petition will fail as his lawyer Taneja was not in a position to defend him in the
Court. Hence, the fundamental right of Gayaram to have a lawyer of his own choice cannot be
exercised.

2
1993
SECTION IV:
Q. No. Answer
1. Yes, an offence is committed by A under the said Act as there was intention on the part of
A to humiliate B who was a member of scheduled caste.

2. Shamanna can seek compensation from Ramanna because though Shamanna’s cattle
trespassed Ramanna’s land but he owed a duty to treat those animals with humanity.

3. Nanjappa should not be prosecuted for culpable homicide as right of private defence
towards property extends to causing death in the cases of theft or mischief.
4. In this case Raghvendra Rao, though was not a partner himself, but he allowed that to be
projected to others. He did not object to his name to be used by Vijayan and also
participated actively in the inaugural function.

5. B can seek an injunction from the Court to restrain A from disturbing classes because, he is
justified in using his house for the classes only for two hours which can be considered as a
reasonable use of his premises.

1994
SECTION V, PART A:
Q. No. Answer Solution
1. (c) According to the principle HMT (master) shall be held liable because the driver
committed wrong during the course of employment.

2. (a) According to the principle P is bound to share the profits as the profits were made in the
course of partnership business.

3. (b) There was no violation of the principles of equality. According to the principle like
should be treated alike. Thus people whose income is more than 60000 are not equal to the
people who earn less than 60000.
4. (b) A will succeed in the suit filed against B as B did not provide old rice which was agreed
between the two.
5. (a) A should be sued because a master is liable for the wrongful acts of the servant and in the
given situation it was N who did the wrongful act during the course of employment.

6. (b) According to the principle X will not succeed because an adopted child shall be deemed
to be the child of the adoptive parents with effect from the date of adoption. Hence X has
no claim in the property of natural parents.

7. (b) Rahim cannot claim the reward as according to the principle he did not have any
knowledge about the proposal printed in the newspaper & hence there was no acceptance
made. Thus Ramesh is not liable to pay the reward.

8. (c) Intention is an important element for theft. In the given question Suresh did not commit
theft as he did not have dishonest intention.
9. (a) R is liable because his actions led D to believe that he is associated with the business of A
& B.

10. (a) The partnership firm is not liable, because the partners had unanimously agreed that none

3
of them would raise a loan in the name of the firm. Despite this fact A took the loan, thus
he is himself responsible to repay the loan to the bank.

SECTION V, PART B:
Q. Solution
N
o
.

1. Not liable as there was no intention on the part of the surgeon.

2. The non compete clause is unreasonable and hence not enforceable under the Indian Contract
Act.

3. X is liable for causing the death of the friend since the act of electrocution amounted to
negligence.

4. This is a valid cause for fraud since the mistaken of identity was intentionally created by B for
the purpose of the procurement of consent for the modification of the mode of payment.

5. The contract is merely difficult to perform and hence does not graduate to the degree of
impossibility and therefore, the contarct is not frustrated.

6. The fundamental right to be represented by a lawyer of one’s own choice does not extend to the
engagement of lawyers who are in custody for breach of law.

7. Under the given principle there shall be no obligation to make the additional payment. Had the
principle been of promissory estoppel, liability to pay would exist.

8. Ramanna would not win the suit since the order is valid under the law.

9. This amounts to false imprisonment since there is no express or implied authorization under the
law to detain a miner within the mine.

10. Double Jeopardy does not apply in this case since the two actions are independent of each other
and the first one does not amount to conviction under the law.

1995
SECTION V, PART A:

Q. no. Answer Solution


1. (c) The driver would loose because he drove without proper headlights and the lady did not
give consent to the injury caused.

2. (c) Bank will be liable for the wrongful act of the manager according to the principle as it
was for the manager to ensure the authentication.

3. (a) According to the principle Gokuldas will be liable, because Veerappa was injured by
Kannappa’s act in the course of employment.

4. (a) According to the principle Hindustan Petroleum will have to pay, because Hanuman

4
struck the match in the course of employment.

5. (c) Kohinoor will be liable because Mohammed was also an employee of Kohinoor and he
did the wrongful act in the course of employment.

6. (a) According to the principle b did not have the intention to kill A hence he is not liable
for Culpable Homicide.
7. (a) A is liable for cheating as he fraudulently induced B by misrepresenting himelf as a
sales tax officer to deliver TV to him.

8. (b) The master of the ship is liable as he was the one who engaged the stevedores. He could
have reasonably foreseen the consequences of his act.

9. (a) B is liable for murder because he gave a severe blow on A’s head and was aware of the
fact that B was in a delicate condition.

10. (c) Sakalchand will be liable because he should have taken note of the adjacent playground
& acted with reasonable care.

SECTION V, PART B:
Q. no Answer
1 This is a valid cause for fraud since the mistaken of identity was intentionally created by
B for the purpose of the procurement of consent for the modification of the mode of
payment.

2 The contract is valid as there is recirprocity of undertakings. Veena has to return the
jewellery.

3 Vaswani shall be eligible to claim compensation for the loss of profits since the same
arose in the ordinary course of things.

4 The damage here was not foreseeable and hence shall not fall within the definition of
“direct consequence”.

5 This is sedititious speech since the form of removal does not specifically refer to
electoral means.

6 Veerappan is not liable for theft since the sandal trees do not qualify as “movable
property” until their severance from the ground. A case for attempt to theft may
however, be initiated.

7 Kumud is not liable for theift as she did not have the requisite dishonest intention for the
offence of theft.

8 Brother-in-law does not qualify as a near relative.


9 This confession does not amount to a legal confession as admissible evidence in a court.

10 Liable for the possession of heroin since the liability laid down under the principle is
strict in nature.

1996
SECTION V, PART A:

5
Q. no. Answer Solution
1. A- Its not foreseeable on the part of ram to know that shyam is carrying
firecrackers.

2. B- It is not foreseeable on the part krishnan to know that a pregnant women


would see the injury and would have miscarriage.
3. B- Anant is not liable for amputation but is liable for the fracture

4. C- Krishnan's sensitivity is brought to the fore as he could not put up with any
continuous sound.
5. B- No recklessness on the part of jaspal as he took precautionary measures. He
would have been held liable for the boy's injury if it had been shown that
children usually play around his house.

6. C- Since he has been paid commission by the bank his swindling of money
comes under the course of employment.

7. B- Ram exercised his discretion when he signed the contract with fixed terms
which speaks of his consent.
8. C- As it was a registered letter the post man was duty bound to enter the
premises
9. A- Since he tolerated them he has given implied consent to them and he would
be held liable.

10. B- Kishanchand has already discharged his duty beyond which he can't be
held liable for the drinking propensity of his friends.

PART B:
Q. No. Answer
1. There is free consent as Deepak could have verified the car using his ordinary due
diligence.

2. Here the college has dominated the will of kiran and obtained her consent not freely.

3. The saree dealer is only liable to pay for the difference in the market prices, not for
the exorbitant price at which it was bought.

4. An agent is not supposed to strictly comply with the instruction of his principal only
in the interest of his principal which was precisely done in this case by storing the
goods in a better maintained warehouse.

5. The two divisions are complete separate though under the same company. Both the
divisions worked independently for two different customer so no conflict of interest
involved. Also, the work performed was done pure merit, therefore, the commission
has to be paid.

6. Law which has enforced DPSP can't be struck down on the ground of revenue
getting affected. Also this is injurious to public health and law and order situation.

7. X's fundamental right of religion and the right stick to tenets therein can't be violated.
Also not saluting the flag doesn't amount to disrespecting it.

6
8. Disinvestment is not the equivalence of depriving individuals of their property, done
only for boosting the economy. Also, the preamble is not enforceable part of the
Constitution.

9. Freedom of religion is a fundamental right and can't be violated by UCC and


therefore has to be struck down.

10. Development should not be harmful and self defeating. Since the power plant will
damage the ecology it is not sustainable.

1997
SECTION V, PART A:
Q. no. Answer Solution
1. A- The house owner has the right to the air column over to a reasonable height of
8/10 feet.

2. C- He attempted to deliver the counterfeit currency of which he had full knowledge.

3. A- Since the dog had already run away using force on it amounted to wrongful use of
rpd..

4. A- As the the inconvenience and illness arose in the usual course of breach of the
contract.

5. B- By not keeping watchman the firm did not take so much care as a man of ordinary
prudence would.

6. A- The employer is absolutely liable in case of injury suffered by his employee


7. C- There was no common intention to commit murder.

PART B:
Q. no. Answer Solution
1. C- There is no criminal wrong on the part of defaulter so only civil action.

2. B- Because it was the liability of Mr Sampath who delegated the driving to Vivek.
3. A- He exposed himself to such risk involved in the game.

4. D- There is an implied contract between the two as Chatterjee consumed the sweets.

5. B- Since the decisions of pollution control by the company are taken by directors,
they can rightly be prosecuted.

6. C- In such a manner as not to cause injury to the interest of other states.

PART C:
Q. Answer
N
o
.

7
1. Since Videocon is the beneficiary of the contract it can rightly sue.

2. Fundamental right is subject to reasonable restriction and this restriction is one such.
3. Establish the omission of the duty to ensure the pupils are within the premises of school on
the part of school.

4. Prosecution has to establish Mr Singh was negligent while driving.


5. The President has to act in accordance with the advice of council of ministers headed by
pm

6. Narendra’s act was mentally cruel as it was in complete disregard of the financial capacity
of Dipti's parents. Therefore, it's a ground for divorce.

7. Actions are not justified as he gone much beyond seizing and detaining the animals by
killing them.

1998
Section V, Part A:
Q. no. Answer Solution
1. B- Bank has projected ananth as it's agent throuhy the acts.

2. B- This was an act performed on personal basis therefore no liability.

3. C- The company would be liable because disobeying will not bring the employee
outside the ambit of vicarious liability.

4. B- The only motivation for buying the jar was its aesthetic properties not the
statements made by zavwrilal.
5. C- The neighbor is sensitive man and the interference was not unlawful.

6. B- Ram's negligence did not deny shyam's title to his bicycle.


7. B- Iin the event of sale the requisite terms have to be applied.

8. B- Since he paid additional two lacs on account of negligence of S.

9. C- Ram never consented to a speed in excess of the limit prescribed.

10. B- Driver was not privy to offence of theft.

1999
Section V, Part A:
Q. No. Answer
1 As per the principle the amendment is not equivalent to law, and when it is not
violative of the basic structure, which right to property is not, it is valid

2 The prohibition will not hold as the proceedings under sea customs and trial for
criminal conspiracy are two different proceedings

8
3 Since probation of offenders act reduces the punishment it can be applied
retrospectively as it is not penal in nature

4 Legal aid is extended to every person, who is not capable of engaging a lawyer by the
state at its expense

5 The opportunity of written submission is as good as the right to being heard


6 Enquiry official did not give sufficient opportunity to the govt servant for putting
forward his defence

7 The father has vested interest involved as it is the issue of his son's admission. So in the
interest of transparency and propriety father should have recluse himself from the
admission process

8 In the given facts there is no scope of subjectivity and miscarriage of justice.


Consequently, the proceedings will not get affected

9 Since A has the opportunity to make his house free by reaping the loan amount it is
valid. And only in case of A's default B has the right to claim the house

10 As an oral test comprises 200 marks and so long the objective parameters are used by
interviewers the interview process is valid

2000
Section V:
Q. no. Answer Solution
1. C- Since neither particular oil nor a fixed price is mentioned, it renders the agreement
uncertain. Hence A and B can be the probable reasons.

2. C- The rule of promissory estoppels:- If on the basis of unilateral favour/promise the


other party does something significant, the first party which made such a promise
shall be liable.

3. C- A commits an offence the moment he takes aim at B. Prior to that point of time,
everything shall come under preparation which is not an offence.
4. A- One cannot get a contract declared void on the grounds of undue influence if he is in
a position to exercise his discretion, or make his choice, or has the option to exit.

5. B- Though B was stabbed repeatedly, he did not die. Instead, he died due to the
negligence of the doctor. Since there was a likelihood of survival even after being
stabbed, A shall be liable for attempt to murder only and not murder.

6. B– It is evident from the facts that A knew the consequences of his actions (since he
told B not to tell anybody about the chain), and hence, he would be held liable.

7. B– As per the principle, husband and wife are not responsible for any wrong to each
other. Therefore, H would not be liable. X would be liable on account of master
servant relationship.
8. C- The offence of mischief is not committed as there is no damage to property. Had
there been any damage, A despite being the owner would have been liable.

9
9. A- Since A has not paid to the dry cleaner, the dry cleaner is said to be in the rightful
possession of the coat. When A takes it away surreptitiously, he commits theft, by
causing wrongful loss to the dry cleaner, even though he is the owner of the
property.

10. B- Principle 1 applies here. Y is liable to compensate only for the rice he has
consumed, and not for the whole bag.
11. C– The hospital authority was justified in giving the information to B, as it was in the
public interest that the information was disclosed. The health of the person was
directly involved as HIV being a communicable disease.

12. C- For the offence of defamation, the defamatory statement must be published, i.e,
communicated to a third party, by the person alleged to be defaming himself. Here
since B, and not A, is communicating the defamatory statement, therefore, there is
no defamation.

13. A- The hospital shall be vicariously liable for the doctor’s negligence as he is a servant.
According to the Principle 2 “a person is a servant of another or not is to be
determined by finding out who controls the method of work or owns the tools or
who benefits by the profit of the venture or bears the loss.”
14. C- There was no duty of care on the part of the stadium owners or the cricketer and
hence, no breach of duty can be attributed to them. Moreover, the incident was also
not reasonably foreseeable.

15. A– As per the Principle 1, the acceptance was complete as the mail was sent and it was
out of the power of acceptor, i.e, A. Because, the acceptance was complete, hence,
binding on both the parties.

2001
Section VII:
Q. No. Answer Solution
21. C- Obalesh took the money in personal capacity and not in the course of
employment. Hence the biopharma would not be liable.
22. B– There was no concensus as idem in the given set of facts as per the principle,
because the main concern of Shakuntala while buying the vase was attracted
towards its aesthetic characteristics and Ganesh Gallery was concerned about its
antiqueness and being unbreakable. Hence, though they agreed on the vase but not
in the same sense.

23. B- By coming to watch the match in the stadium, Vijay had agreed to all attendant
risks that could be reasonably foreseen, which includes the risk of being hit by the
ball. Hence the BCCI or Sachin would not be liable.

24. A- The prerequisite for murder is the intention of causing death which is clearly
missing here. Therefore on account of absence of mens rea, C cannot be made
liable for murder.

25. A- Here A by falsely representing himself as the commercial tax officer induced B
to deliver the microwave at an installment basis. Had he not posed as the

10
commercial tax officer, B would not have delivered the same on installment basis.

26. C- Since Dinesh did not act with diligence and to the best interest of the principal;
he did not completely perform the functions of the agency and hence would be
liable.

27. B- Since sufficient warning had been given about the electrified fencing, this is not a
case of causing willful harm or reckless disregard.

28. A- A person is liable to everyone who is reasonably foreseeable to be affected by his


negligent action. Here, too by not closing the door Basappa breached the duty of
care. Hence would be liable.

29. B- Since the mode of removal of ruling party is by lawful means, that is, by voting,
it is not sedition.

30. C- Since, Tataiah, being the owner allows Jaggesh to drive the car, he would be
liable.

31. B- It is unclear as to which horse is to be sold, therefore, the agreement is uncertain


and parties are not bound by the agreement.

32. A- By actively concealing the cracked hoof, and by not denying that the horse was
sound on being asked, A as he has led B to believe that the horse was sound.

33. A- As his profits were earned during the course of partnership business, he is bound
to share the same.

2002
Section IV:
Q. Answer Solution
No.
111. A - The student had no intention to cause harm and hence not liable for battery.

112. A – The owner shall be held liable for causing harm in the excess of self defence because as
the police arrived and announced to stop firing, the owner should have stopped.

113. B - Since the father of the child, announced the reward after the person went in to save the
child, it can be concluded that he acted on his own.

114. B – As the spleen was removed, it is not a property of the person now.

115. C - The scope of an agent’s authority is conditioned not only by the powers conferred by the
principal, but by the reasonable public perception of such authority.
116. C - Rajaram and Cambridge Academy being two independent parties, the Academy cannot
be said to dominate Rajaram and thereby to induce him to enter into a contract. Secondly,
Rajaram himself agreed to the terms of the admission without any compulsion, hence there
is no element of undue influence at all.

117. B - The contract is void on account of being uncertain.

118. B - Nobody shall make use of his premises in such a way as to damage the neighbour’s
interest in the latter’s premises.

11
119. B - The consequence is not forseeable by a reasonable man.

2003
Section IV:
Q. Answer Solution
No.
132. D - Purchasing a ticket and being a spectator (in the stadium) is an implied consent
to any harm caused.

133. B – Master servant relation existed between driver and B.

134. A - The purpose was illegal but the bank can successfully plead for the recovery of
loan.

135. D - One has to pay for being a beneficiary of the contract in order to avoid unjust
enrichment of any goods or services.
136. B - Mere quotation of the price doesn't amounts to an offer.

137. C - When an agent appoints another action within the authority, the other is
responsible to the Principal
138. B – Balram’s liability only arises towards Antony as an ordinary prudent man. He
cannot be held liable for a mystery disease as that was not reasonable for a prudent
man to foresee.

2004
Q. no. Answer Solution

116. C Option (c) is technically correct as compared to option (b)

117. B Strict Liability applies Municipal corporation entitled to recover

entire cost.

118. B along with the explanation in (b), the threat had come to an end since

the robbers were fleeing.

119. B half furlong is reasonable distance hence no contributory negligence.

120. B Flight attendants were unaware of his situation.

121. B Doctrine of ostensible employment. Kishanlal must have taken

adequate steps to inform the clients about Kiran's terminations of

employment.

12
122. C the adjoining circumstances should have been taken into

consideration. Children in the vicinity did not warrant such extreme

measures.

123. B Manufacturer's liability, Kumar cannot be expected to know what

was inside the thread.

124. A authorized act done wrongfully & BTS is an independent contractor

here therefore only BHEL to be deemed as master.

125. B Collection outside the bank comes outside the course of

employment.

2005
Q. no. Answer Solution

89. B straight forward Principle facts application based question.

90. A Jewelry does not constitute house hold necessity.

91. A Mr. Rajesh Chawla is not at fault at all.

92. C Straight forward principle fact application based questions.

93. A Straight forward principle fact application based questions.

94. B No dishonest intention.

95. B She has sufficient income to maintain herself.

96. D requirements laid down in principle have been met.

97. D she would get the maternity leave after producing the certificate

2006
Q. no. Answer Solution

93. C Straight forward principle application based question.

94. A his rash and negligent act has endangered human lives.

95. C Regardless of the compulsions the crime of bigamy has been

13
committed.

96. A Year of examination is 2006, 12 years hence lapsed therefore (a).

97. B the mandate of the arbitrator has come to an end

98. B Confession is accepting the commission of crime by oneself.

99 D Computer cannot be treated as another person.

100. C The contract is void as it is in restraint of marriage.

101. A Debt is time barred.

102. B Agreement is void since it is contrary to law.

103. C Straight forward Principle Fact based question.

104 B Straight forward Principle Fact based question.

105. C The liability of the employer is absolute as per the principle.

106. B Only 4 persons turned up.

2007
Q. no. Answer Solution

113. D There was a duty in tort law irrelevant of whether or not the contract

existed.

114. C Straight forward application of Principle to facts.

115. D Z’s act caused inconvenience

116. B liable for ordinary damages only

117. B University negligent and no contributory negligence on the child’s part.

118. D option to be amended he has to pay the towards maintenance.

119. E Act of God outside human control.

120. B Education to come under contract of necessities.

121. A Object of the contract i.e. to make country made pistols is unlawful.

122. A horse betting exception to wagers.

14
123. C Vicarious liability since work was done in the master’s course of

employment.

124. B Terms of contingent contract not met.

125. A Snake charmer to compensate for his careless act

NALSAR
2000
Question Answer Explanation
number
116 A According to the principle Kohinoor (master) will be liable because
Samuel (servant) acted negligently by handing over the steering to
Mohammad during the course of employment.
117 C According to the principle intention is important. B is not liable for
murder because he lost his self control and did not have the intention
to kill
118 D The runner cannot claim damages because firstly, he had voluntarily
consented to the risk involved by participating and secondly, it was
not reasonably forseeable.
119 D The buyer will not succeed because A is a minor and thus not
competent to enter into a contract.
120 C The landlord would fail because he did not pay X for 12 months
which was a form of bonded labour and against public policy.

2001
Question Answer Explanation
number
116 D Because she did not have knowledge that the wounded mam is the
enemy general, and she treated him as an ordinary patient of hers.
117 C When the robbers were fleeing from the scene they no longer put X
under any threat. So, X's firing on the feeling robbers and killing if
one them us certainly the use of more force than required.
118 A The vagabond got killed by W in the ensuing struggle to protect
herself, so the act was completely in the exercise of private defence
as initially she did raise the alarm but to no avail.
119 D Because B's consent was not freely exercised.
120 C The agreement is void on account of being unlawful and against

15
public policy as it was about getting the admission done on the basis
of influence and not merit.

2002
Question Answer Explanation
number 8
(76) B Please see Section 17 of the Indian Contract Act, 1872.

(77) C Please see Section 31 of the Indian Contract Act, 1872, which defines a
contingent contract and read the same with Section 32 which
specifically, states that contingent contracts are not per se void.

(78) C Reciprocity is essential for the validity of a contract under the Indian
Contract Act, 1872. Please see Section 50 of the Indian Contract Act,
1872.

(79) B Please see Section 2(d) of the Indian Contract Act, 1872.

(80) A This is the basic principle of “Privity of Contract”. However, there are
certain exceptions like beneficiary etc.

(81) C While there are certain federal features under the Constitution of India,
1950, there are certain overlapping powers as well. Further, the
Parliament exercises higher jurisdiction in the concurrent list as well.
The distributions of power are specifically set out in Part XI of the
Constitution.

(82) A Whilst there are some negative rights like Right to Life under Article
21, most of the Fundamental Rights set out in Part III of the
Constitution of India are positive rights.

(83) D Right to property was covered under Article 19(1)(f) earlier, however,
the same was removed through the 44th Constitutional Amendment,
1978 and transferred to Article 300A of the Constitution.

(84) B The initial draft of the Constitution was 395 Articles on the date of its
adoption on November 26, 1949. As of 2016 the Constitution of India,
1950 comprises 448 Articles.

(85) B Please see Article 21 of the Constitution of India, 1950.

(86) A The spelling with ‘e’ is British, the spelling with ‘i’ is North American.
The same goes for the nouns, “inquiry” and “enquiry”. There are of
course other differences in spelling between American and British
English.

(87) B There are some exceptional circumstances like under Section 190 of the
Criminal Procedure Code, 1973, where the complaint can be filed before
a magistrate in cases of inaction by the police.

16
(88) A Chipko movement started in 1970 by Sunder Lal Bahuguna.

(89) B Any waste which is generated during the diagnosis or immunisation of


human beings or animals or in research activities pertaining thereto.
(90) B Age for marriage as per Prohibition of Child Marriage Act 2006 is 21
years for Grooms & 18 for bride.
(91) B Refer to Art 76 of Constitution of India.
(92) C The allocation of seats is made on the basis of population of each state.
Refer to schedule 4 of the Constitution of India.
(93) A Vice President is the ex officio chairman of Rajya Sabha.
(94) D On a similar note, the writ jurisdiction of the Supreme Court of India is
parallel covered under Article 32 of the Constitution. Please note that
the writ jurisdiction of the High Court under Article 226 is wider than
that of the SC under Article 32. This is because the jurisdiction under
Article 32 is limited to violations of Fundamental Rights alone, whereas
under Article 226, one can file a writ petition for violations of
Fundamental Rights or any other Legal Rights.

(95) C The primary purpose of establishing a judiciary is ultimately the


adjudication of disputes between parties, through appropriate
interpretations of the law and further clarifications of the meanings of
the various legislations.

(96) B There are various legislations which deal with insolvency and
bankruptcy in India, including the Provincial Insolvency Act, 1920.

(97) D Know-how and discovery neither are patentable under Indian Law.
However, know-how may be protected by copyright laws, whilst
discoveries cannot be protected under and Intellectual Property
Legislation in India.

(98) C Please see Article 32 and Article 226 of the Constitution of India, 1950.

(99) B The written Constitution of India, 1950 was adopted on November 26,
1949 and came into effect on January 26, 1950.

(100) A Please see the Trade Marks Act, 1999.

(101) B The principle directly states that the dangerous activity must be carried
out subject to the knowledge about the presence of a trespasser. Since
‘A’ did not take the same into account, ‘A’ shall be held liable.

(102) D Military truck on duty to check on army personnel at different posts is


clearly a sovereign function.

(103) C Direct application of Kasturilal v State of Uttar Pradesh,AIR 1965 S.C


1039.

17
(104) C Ultimately, ‘B’ had the last opportunity to avoid the accident and hence
there will be liability based on the direct application of the principle.

(105) D There is essentially a promise to refrain from doing something that ‘A’
had a legal right to do which is a negative consideration.

(106) C Officially termed as The Constitution (Sixty-first Amendment) Act,


1988, it was the 61st Amendment that lowered the voting age of
elections to the Lok Sabha and to the Legislative Assemblies of States
from 21 years to 18 years.

(107) D Please see the Rules of Procedure and Conduct of Business in Lok
Sabha, Fifteenth Edition, 2014.

(108) D Please see Article 75(5) of the Constitution of India, 1950.

(109) D This call was given to the INA Army to motivate them .
(110) D It was appointed by Lord Rippon on primary education.
(111) C It was created in 1997.
(112) A Please see the explanation for Q. 84 above. Right to property was
covered under Article 19(1)(f) earlier, however, the same was removed
through the 44th Constitutional Amendment, 1978 and transferred to
Article 300A of the Constitution.

2003
Q. No. Answer Explanation

(76) C Please see Section 73 of the Indian Contract Act, 1872.

(77) B Please see Section 15 of the Indian Contract Act, 1872.

(78) B Please see Section 73 of the Indian Contract Act, 1872.

(79) C Please see Section 23 of the Indian Contract Act, 1872. An agreement
to share the benefits of public office is against public policy.

(80) B Please see Section 30 of the Indian Contract Act, 1872.

18
(81) B The Constitution of India came into force on 26th January, 1950.

(82) A The Preamble further underwent very famous changes through the
42nd Constitutional Amendment, 1976.

(83) D As of 2003, the correct answer would be 26. However, currently the
total number of judges of the Supreme Court has increased to 31
including the Chief Justice of India..

(84) C Westminster model of democracy is basically the British


Parliamentary model which has been adopted by India.

(85) C The 42nd Amendment changed the description of India from a


“sovereign democratic republic” to a “sovereign, socialist secular
democratic republic”, and also changed the words “unity of the
nation” to “unity and integrity of the nation”.

(86) D Actions regarding criminal proceedings are taken by the state.

(87) B Please see Section 93 of the Indian Penal Code, 1860.

(88) B Please see Section 120A of the Indian Penal Code, 1860.

(89) C Please see Article 72 of the Constitution which deals with


Presidential pardoning power. Further, see Article 161 which deals
with the pardoning power of the Governor.

(90) B Please see Section 441 of the Indian Penal Code, 1860.

(91) D The current strength of UN is 193.

(92) D ICJ was established in the year June 1945.


(93) A Theme for World AIDS day is “Hands up for# HIV Protection.”

19
(94) B UN was created in 1945.
(95) D The names of Judges in ICJ are Nagendra Singh, R S Pathak,
Dalveer Bhandari.
(96) D Please see Part III of the Constitution of India, 1950.

(97) C There were 7 members in the drafting committee.


(98) A Self explanatory
(99) B For example please see Section 13 of the Hindu Marriage Act, 1955.

(100) B For example please see the Hindu Succession Act, 1956.

(101) C India’s first nuclear test was conducted on 18th May 1974, also known
as smiling Buddha (Pokharan 1)
(102) B BCCI instituted this committee in June 1997.
(103) C The Prevention of Terrorism Act, 2002 (POTA) was an Act passed by
the Parliament of India in 2002, with the objective of strengthening
anti-terrorism operations. The Act was enacted due to several terrorist
attacks that were being carried out in India and especially in response
to the attack on the Parliament.

(104) A Self explanatory


(105) A POTA 2002 was passed by joint session of Parliament.
(106) B She was the first Indian born women and second Indian person to fly
in space.
(107) B He scored 673 runs and 2 wickets.
(108) A It is a serious form of pneumonia caused by virus.
(109) A Operation Iraqi freedom started in 2003.
(110) D Self explanatory
(111) A M M Panchi commission also dealt with central state commission.
(112) B Only two non Indians Khan Abdul Gafar Khan & Nelson Mandela
are the recipiants of Bharat Ratna Award.
(113) A The current chairperson of NCDRC is J. DK Jain.

20
(114) C Self Explanatory.
(115) C Self Explanatory
(116) D Being a peon, Amit cannot be held liable for the frauds done by the
finance company. Not a slight indication of Amit being involved in
the wrongful conduct of the business.

(117) B Rod is an article which is not primarily adopted or designed as a


weapon.

(118) D There was no consideration and the exception doesn’t cover a


mistress.

(119) C He clearly did not provide the information voluntarily.

(120) D Any agreement will be void in the absence of free consent.

2004
Question Answer Explanation
number
51. C Art. 352 of the COI
52. B Art. 74 of the COI
53. D art. 75 of the COI
54. C right to freedom of speech and expression deals with censorship
55. C art. 80 of the COI
56. C art. 56 of the COI
57. C panchayti raj is system of self-government
58. A art. 124 of the COI
59. A The Consumer Protection Act 1986
60. A the statement was made because our Supreme Court besides being
a federal court acting as a guardian to the constitution also
exercises appellate and advisory powers

21
64. A excise duty is a form of indirect tax
73. A rights that can be made enforceable in a court of law
74. D all three i.e. existence, breach as well as resulting legal injury have
to be present
75. D all of them had knowledge of the defamatory content
76. B as per Prohibition of Child Marriage Act,2006
77. A maintenance to a muslim wife after divorce
78. A refer to Limitation Act, 1963, Section 2(m)
79. C intention relevant only in some torts like malicious prosecution
80. A refer section 499 of the IPC
81. B section 120A of the IPC
82. D section 302 of the IPC defines punishment for murder
83. A Cognizable offences are covered under section 2(c) of the CrPC
84. B Pota was one of the acts passed after a joint sitting of the
parliament
85. C past consideration is the price said or service rendered at the desire
or request of the promisor followed by a subsequent promise
86. B The offer of 45000 made against the original offer of 50000
amounts to a counter offer
87. D display of goods and enquiring shall qualify merely as invitation
88. B agreements with future consideration are not necessarily invalid
89. D The doctrine of frustration of the Indian Contract Act
118. A Bobby is guilty since he had the understanding of what he was
doing
119. B act will come under the course of employment
120. B the removal of the government was sort through lawful means

2005
Question Answer Explanation
number
35. C Art. 53, COI
36. D Art. 64, COI
37. B Art. 75 of the COI, no fixed tenure since they enjoy office at the

22
pleasure of the president and can be removed at any time upon the
recommendation of the PM
38 A art. 368 of the COI, however the basic structure cannot be
infringed
39 C Made a legal right via 44 th Amendment to COI, u/a 300a now
40 A Art 60, COI
41 C enable villages with a self-government
42 C art. 124(2) of the COI
43 B Income Tax Act, 1961 for levying direct taxes
44 B Independence of judiciary and judicial review borrowed from the
US constitution. This concept was enunciated by Justice Marshall
of the US Supreme Court in the case of Marsbury V. Madison
46 A tax based on value of goods is ad valorem tax
48 C This act was passed in the year 1999.
57 B vicarious liability wherein the master is held liable along with the
servant if the work done by the servant is under the due course of
employment
58 D The right to private defence available under law in order to protect
one’s life and property and the life and property of others
59 B slander is defamation in an oral/verbal form.
60 B ‘Nikah’ is a civil contract as per Sharia Law, refer case Abdul
Kadir v. Salima
61 A refer case Seems Ashwini Kumar v. NCT of Delhi
62 A damages are unliquidated i.e they are not pre-certain or decided
before hand.
63 A Law of torts is not codified as such and has developed through
judicial decisions.
64 C Innuendoes are words with hidden derogatory meaning.
65 A the attempt to commit a crime is punishable since injury, either
mental or physical is caused from the stage of attempt.
66 A sex determination banned in India in 1994 via this act67. a- refer
section 320 of the CrPC
68 D Advocates Act, 1961, minimum age required to be an advocate is
21 years
69 C damages awarded on the basis of proving the exact loss suffered.

23
70 A In case of a breach of contract the party can demand ordinary
damages
71 A non-gratuitous benefits have to be compensated in order to avoid
unjust enrichment, section 70 of the Indian Contract Act
72 C Void as the object becomes unlawful in nature, section 23 of the
Indian Contract Act
73 A damages given to set an example are known as exemplary
damages
102 A the property was taken away from the rightful possessor without
his consent in case of non-payment, amounting to theft.
103 A it was negligence on his part to leave the door open, therefore
liable.
104 A Ghantewala will get the damages since it being new year’s, time
was the essence of the contract.

2006
Question Answer Explanation
number
91. B There is no remedy for Ashok because he called 100 for getting
the flight delayed and not any genuine emergency. He did this act
for his personal convenience.
92. A confession made in Court is always free and voluntary. As Amit
confessed in the Court, so there is a likelihood that he might be
punished.
93. B During driving, trying to overtake amounts to rash and negligent
act. Hence, liable.
94. B Driver is liable because he should have been more careful while
driving an unmanned railway crossing. Moreover, his brakes were
not working properly so that amounted to negligent act.
95. A A is liable for rash and negligent act as suddenly turned the car in
order to reverse the direction. This act ultimately led to series of
collisions.
96. D Driving at the speed of 80 km/hr whose brakes had become
dysfunctional is a rash and negligent act.
97. B Leaving lions to check trespass is again a rash act and hence the
circus owner shall be liable.
98. B Chairman is liable because he ought to have put a warning board
for the trespassers that wires are electrocuted
99. B Yes he is liable because as per the principle the transfer of
property for consideration paid by another is allowed only in the

24
favour of wife or a daughter.
100. B No, the agreement is not void as inadequacy of consideration is
not a ground for rendering the agreement void.
101. D Refer to Article 20 (2), Constitution of India, 1950.
102. B Sec 120B of IPC
103. D Self Explanatory
104. A Self Explanatory
105. A Self Explanatory

2007
Question Answer Explanation
number
52. B Mortgage is always of immovable property and pledge and
hypothecation of movable property.
53. B It is an expression of willingness to negotiate. A person making
an invitation to treat does not intends to be bound as soon as it is
accepted by the person to whom the statement is addressed." A
contract is a legally binding voluntary agreement that is formed
when one person makes an offer, and the other accepts it.
54. A Civil law, civilian law or Roman law is a legal system originating
in Europe, intellectualized within the framework of late
Roman law, and whose most prevalent feature is that its core
principles are codified into a referable system which serves as the
primary source of law.
55. C It means repairs.
57. C Refer to Section 125, Crpc, 1973.
58. D Refer to Section 2 of Crpc, 1973.
59. D Unliquidated damages are such as are not yet reduced to a
certainty in respect of amount, nothing more being established
than the plaintiff's right to recover; or such as cannot be fixed by a
mere mathematical calculation from ascertained data in the case.
60. B 42nd Constitutional Amendment Act, 1976 is also referred to as
Mini Constitution.
61. B An act which can be rescinded at the option of the either party is
known as voidable.
62 C The socialist pattern is established by Part IV of the Constitution.
63 C When two or more persons, by fighting in a public place, disturb
the public peace, they are said to "commit an affray".

25
66 C it is a first ever document on Fundamental Rights.
67 A Confession of an accused is relevant and admissible when made
in the custody of a magistrate. Section 164, Crpc, 1973.
70 A Sir Ivor Jennings remarked Constitution of India as a Paradise of
Lawyer’s.
74 C Anyone living in the area, who is an adult, that is 18 years old or
more, is a member of Gram Sabha.
76 B Husband and Wife are treated as one for the purpose of Civil Law
and not Criminal Law.
80 B Contempt of Court is of two types: Civil and Criminal Contempt.

NLU-D
2008
Question Answer Explanation
number
(1) C Self explanatory
(2) D Apart from all of these, the Supreme Court of India also enjoys
writ jurisdiction under article 32 of the Constitution.

(3) B Only a Minister or a Private Member can introduce a Money Bill


(4) B Self explanatory

(5) B The US Constitution is Federal, thereby specifying only Union's


subject matters and leaving all other matters to the States.
(6) C
(7) D
(8) C Lahore session of December 1929, Congress passed the Poorna
Swaraj resolution.
Nagpur Session 1920 At the Nagpur session in December 1920,
some crucial changes were made in the organization of the
Congress, so that it becomes a real political party.
Madras Session- India would be given Dominion status. This
means independence within the British Commonwealth.
Calcutta session- Return of Gandhi
(9) C The Innuendo
A statement is prima facie defamatory when its natural and
obvious meaning leads to that conclusion. At times though a
statement may be prima facie innocent but some latent or
secondary meaning may be considered defamatory, the burden to

26
prove the latent meaning lies on the plaintiff. For ex- Saying
even A is an angel may be slander if the statement was
understood to refer to a criminal gang known as “The Angels.”
(10) A Article 41 directly deals with right to work under the
constitution. Please note that this is a directive principle of state
policy and therefore not actionable but the state can make law in
compliance with the same.

(11) B Although there are certain exceptions in terms of protocol and


order of precedence even for the Prime Minister, the others
directly have immunity from most criminal prosecutions.

(12) B Directly covered from Article 1 of the Constitution.

(13) D Right to Freedom under Article 19 is very wide in its ambit and
hence has been the subject of the maximum controversy and
litigation.

(14) B Added by the recommendation of the Swarna Singh Committee.

(15) C Added by the 73rd Constitutional Amendment Act, 1992 and


created Part IX of the Constitution.

(16) A Directly covered under Article 75(1) of the Constitution added


by the 91st Amendment and fixed at 15% of the total members of
the Lok Sabha.

(17) C Please refer directly to Article 143 of the Constitution.

(18) B Special leave is granted as a discretionary power of the Supreme


Court. It is granted by the Supreme Court only in exceptional
circumstances.

(19) D Unliquidated damages are basically damages which are not fixed
and liquidated damages are the ones which are pre-determined as
in contract breach cases.

(20) D Assault and nuisance are both torts and crimes covered under the
IPC.

(21) B There is no direct physical contact.

(22) C Articles on display are invitations to offer.

(23) B Idols are juristic persons like companies.

(24) A Judicial Custody is different from Police Custody to the extent


that police custody is essentially is police lock up whereas the
other is sent to jail. Directly under CrPC.

27
(25) C There are some torts like malicious prosecution where intention
is relevant even for Torts, but generally, torts do not have mens
rea as an essential condition.

(26) B Directly under the Indian Partnership Act.

(27) B Article 13 of the Statute of the ICJ.

(28) B Referendum. Referendum is a Latin word, but its modern


meaning only dates from the 19th century, when a new
constitution adopted by Switzerland stated that the voters could
vote directly on certain issues. Thus, a referendum is a measure
that's referred (that is, sent on) to the people.
(29) C The manusmriti is the oldest codified Hindu Law.

(30) D Wakf Boards to be more specific are the religious organisations


within Islam that regulate family disputes in general.

(31) C Since the maximum cases are naturally against the government,
the cause is also the government.

(32) A Haryana
(33) B Under Muslim Law, only the male continues to have a valid
right to divorce except in very specific circumstances.

(34) B Adultery is not punishable for women.

(35) D Refer to Section 125 of the CrPC.

(36) D Direct application of meaning.

(37) A Direct application of cheating since there was intention.

(38) C Extradition essentially means that the countries mutually agree


to deport criminals to each other’s states.

(39) D The Gulf war


(40) C Directly covered under the United Nations Convention on the
Law of Seas – III, 1982.

(41) A Admited to the Inner Temple 1888; call to the bar 1891;
disbarred 1922; reinstated 1988
(42) A It is not reasonable apprehension for the father to assume that his
friends would be kidnapping his daughter. Anyways the reaction
was disproportional too.

28
(43) C In the natural course of things, the effective damages would have
been Rs. 5000/- since the iron was available at Rs. 105/- per
tonne.

(44) B The consent is assumed since the contract was as a consequence


of government order.

(45) A There was no reckless disregard since adequate measures were


taken.

(46) A Stephen would qualify as an agent but not as an employee.


Therefore, if the principle was of agency and not employment,
then there would be liability on the part of the bank.

(47) D There was no subsequent ratification by the surviving party.

(48) D Ignorance of law does not apply in this case since this is a
mistake of law which is not a defence.

(49) D The postman qualifies as a visitor since the letter was registered
and the post box outside aspect is purely to misguide.

(50) C This is directly “sovereign function” as per Kasturilal v. State of


Uttar Pradesh

2009
Q. No. Answe Explanation
r

(46) A Direct application of the definition under the Sick Industrial


Companies (Special Provisions) Act, 1985.

(47) A Economic Problems of South Asian countries.


(48) D Wealth tax on agricultural land was removed in 2013 and
wealth tax itself was completely removed in 2015 by the Union
Government.

(49) D Indian Council is not needed to advise the Secretary of State.

(50) B
(51) A The first draft had only 315 Articles and 8 Schedules. The
question of course does not make it clear but the assumption has
to be about the first since the final draft obviously had 395
Articles.

(52) D In 1935.

29
(53) C The Interim Government of India was formed on 2 September
1946 from the newly elected Constituent Assembly.
The Congress held a large majority in the Assembly (69 percent
of the seats), and the Muslim League held nearly all the seats
reserved in the Assembly for Muslims. There were also
members of smaller parties, such as the Scheduled Caste
Federation, the Communist Party of India and the Unionist
Party.
(54) B Direct application of Article 115(1)(a)

(55) D Popularly known as the Indira Gandhi Emergency.

(56) B Direct application of the Citizenship Act, 1955, Schedule – III.

(57) B Right to Property was a Fundamental Right under Article


19(1)(f) but was removed by the 44th Amendment, 1978 to
Article 300A of the Constitution.

(58) D DPSPs are classified into three parts (a) Gandhian; (b) Socialist;
and (c) Western Liberal.

(59) B Fundamental Rights like 20 and 21 cannot be suspended


because of Article 359 and financial emergency is referred to
under Article 360.

(60) A Must be confirmed by the High Court Divisional Bench i.e. 2


judges.

(61) B Rules of Parliamentary Procedure, Article 198 (2).

(62) B Under Article 141 of the Constitution, only the orders of the SC
is binding on the lower courts but the High Courts only have
supervisory jurisdiction under Article 227 over lower courts
within its jurisdiction.

(63) B Insanity is a general defence under the IPC which covers


irresistible impulse.

(64) A Asiad Workers Case also in detail referred to as the People’s


Union For Democratic Rights & Others vs. Union of India &
Others (AIR 1982 SC 1473).

(65) A Under the Indian Contract Act, 1872, for a contract to be


complete, the acceptance must have reached the acceptee.

(66) C Ramesh is clearly in a position to dominate the will of his


servant. These kind of relationships are generally referred to as
“fiduciary relationships”.

30
(67) A Advertisements and notifications are the most basic forms of
“invitation to offer”.

(68) B Contracts are defined as agreements enforceable by law.

(69) B Frustration of contract does not include commercial difficulty.

(70) D Direct application of Article 352 of the Constitution.

(71) C Voices recorded on a gramophone are only addressed to ear


whereas the others are not.

(72) A Directly covered under the CrpC. For the application under the
Constitution prosecution and punishment both are required.

(73) D Direct definition of colourable legislation.

(74) C The highly controversial amendment was ultimately struck


down by the Supreme Court.

(75) C The Janata Party government headed by Morarji Deasi


constituted an enquiry commission to inquire into all the
excesses committed in the Indian Emergency. It was headed by
Justice J.C. Shah, a former chief Justice of India.

The commission published its report on the illegal events during


the emergency and the persons responsible in three volumes
totaling 525 pages.

(76) A Please note that even the banking holidays are not declared
under the Banking Regulation Act.

(77) B Directly under the Indian Partnership Act.

(78) B Iddat period is basically three successive Tuhrs.

(79) C An approver is usually granted a much more lenient sentence in


return for the evidence. There is no direct definition but as an
instance the Delhi High Court has referred to it hereunder.

http://delhihighcourt.nic.in/writereaddata/upload/CourtRules/Co
urtRuleFile_6G38F1T4.PDF

(80) D Directly covered under the Motor Vehicles Act, 1988, Section
140.

(81) C Although there is no direct definition of the word “battery under

31
our legislations, the general understanding is the same as
criminal use of force as used in the IPC, 1860.

(82) C Directly to be derived from the ordinary definitions of the


words given in the question.

(83) D Retirement age for the High Court Judges is 62 while for the
Supreme Court there is an extension of retirement age by 3
years.

(84) D Direct application of the Directive Principle of State Policy.


Please note that although one cannot file a case for the
enforcement of such a right, the state can anyways make a law
in furtherance of the same.

(85) D The definition for the same is directly given under section 299
of the IPC.

(86) D None of the defences set out herein are valid defences.

(87) C Laloo has in fact granted implied authority to everybody for the
use of the road.

2010
Q. Answer Explanation
NO.
71. D The above mentioned facts state that B’ never had the intention to hurt his
friend and he was only playing a joke and he never used the required force
which would make him liable for the offence of Battery or Assault.

72. C A’s Moment was not completely stopped or restrained as required by the
principle.

73. B There is a lawful excuse with the policeman who was trying to catch a
suspected criminal.

74. A The moment when the clerk read the letter it is considered to be published
and the libel was complete.

75. C B was working under the control and direction of A.

76. B A only authorized the conductor for cleaning the bus or for issuing ticket
and never authorized him to drive the bus. So when he caused the accident
he is liable.

77. C The act of B’ by stopping the truck and giving the lift is an unauthorized act.

78. B Though B was an employee of the bank, he was not authorized to collect the

32
money from the bank. Hence, bank is not liable.

79. C A, B, and C had decided to commit burglary not murder.

80. A A’ will be liable as the direct consequence or effect of putting someone on


the foothills at that time when the temperature is one degree would be
hypothermia. So it could be said that it was reasonably foreseeable and the
person is therefore liable.

81. C Since the height of 6 feet is not sufficient air space in consideration of the
problem. Therefore A will not succeed.

82. C A is liable for trespass, because when the show ended, he was not authorized
to stay in the theatre.
83. A Since surgeon has breached the duty and the moment he does that
negligence is complete.

84. A In the present fact the contract is not complete as the acceptance to the
contract never reached to the offeror.

85. C B posted the letter but the contract would be complete when A receives the
letter and reads it.

86. B A sees the mail and reads the same, hence, it is notified. Also contract is
incomplete.

87. A No force or compulsion was used and the consent given for examination was
free consent. So it is a valid contract.

88. C As both the parties were under mistake of the actual price of the property
and therefore the agreement was never complete and it was a void
agreement.

89. C The object or purpose of the contract is against the public policy and morals.
As getting married in return of the money is not correct morally and also it
is a crime in the law. So the contract is not valid contract.

90. A The purpose of the agreement in the facts was to defeat the provisions of
Criminal procedure code by ensuring surety of person’s guarantee not
achieved here. So it has defeated the provisions of the Law.

91. C In the present facts by entering into the contract, couple disentitled there
children from inheriting the ancestral property and right over ancestral
property cannot be taken away or limited. Hence. the contract
entered to do the same is not a valid contract.

92. B The bidders by entering into the contract wanted to cause loss to the BCCI
which is against the principle of bidding hence, against the public policy.
Therefore, the contract entered is void.

33
93. B Contract is void as contract is restricting company in there lawful trade.

94. A Contract is valid as it does not absolutely restrict the party to approach to
any court; rather it just restricts the party to go to the other court.

95. A As the property an important part of the contract and has not been described
briefly. Also the
certainty to the property is missing in the contract so it is not a valid
contract

96 B The intention at the time when A took back the watch was not dishonest. He
neither caused wrongful loss to B or wrongful gain to himself. So he is not
liable for theft.

97. B Theft is done when something is taken out of possession without that
person’s consent, but breach of trust is done when someone gives the
property in trust and other misuse or misappropriates the property against
trust created. Since, he has the possession of the property; hence, it is a case
of breach of trust.

98. B It is clear by application of the principle that threat and fear were used for
getting the money. Hence, it is extortion.

99. B He committed theft only and not robbery as he moved the property without
using force.

100. A As soon as he moved the wallet, theft was complete. He did not use the
force while moving it but afterwards, hence no robbery was committed at
the time of moving the property.

101. B A is liable as he knew that his act is likely to cause wrongful loss and
damage to the property and therefore it is mischief.

102. B He possessed the knowledge that his action is likely to cause destruction of
the property and therefore he is guilty of mischief.

103. A His intention was to remain there unlawfully and to commit an offence of
theft.

104. B In both the cases facts are same and suit cannot be initiated again on the
same fact.
105. Principle is not applicable on the given set of facts.
None
2011
Question Answer Explanation
number
71. B commission makes them the agents of the bank, hence liable

34
72. D There was no consensus ad idem making the contract void
73. A Ram’s act led to the stealing
74. A the amount spent should be compensated
75. B driver not a party to stealing
76. C The matter trifle in nature
77. C limitation period starts again from the point of part payment
78. C ignorance of law no excuse
79. C Straightforward Principle Application based question. Although Vasan
does not have an unqualified right to lease the house to anybody,
leasing it to an AIDS patient is not unreasonable interference with
others’ right.
80. B half a furlong is reasonable distance
81. A No prohibition under the law to keep a ferocious dog
82. A no compensation as the other party is not at fault and Rajinder Singh
violated the traffic rules
83. A liable to pay 40000 as per the principle
84. C no right to private defence as there was no imminent threat to life
85. A Mangeshkar’s act endangered human lives
86. C Computer will not be deemed as a person for committing the offence
of criminal conspiracy
87. C manufacturers liable as there is the duty of care
88. B university had a duty of care towards the child and the injury caused
was foreseeable
89. A The snake charmer’s act was careless and he owed a duty of care
90. B marriage happened after 6 months which made B’s claim invalid
91. B Under DPSP the state has the right to regulate and organize animal
husbandry
92. C Singing the national anthem is not mandatory as it does not
tantamount to disrespect as held in the Bijoy Manuel Case
93. B Freedom of Expression does not authorize any person to defame the
other
94. A The injury was caused due to Y’s negligence as he did not take
reasonable care

35
95. D settled law in the case of Donoghue vs. Stevenson
96. C Joint liability exists in torts just as much as in partnership law, contract
law, corporate law etc
97. C The second blow was disproportionate and not warranted.
98. A Pvt defence is only available against the aggressor
99. C The threat had come to an end and he must have informed the police
100. B Both of them conspired to poison Z
101. D X and Y are to be punished since they conspired to do a wrongful act
102. C refer to options (a) and (b), also 120A IPC for more on criminal
conspiracy
103. C In this case, X contributed to his own death therefore Y is not liable.
See also Srinivas v. State of Karnataka
104. B The rescue was not performed at the instance of A, therefore the
consideration does not flow from the latter to the former
105. C The agreement was signed under threat of suicide therefore hit by
coercion

2012
Q. No. Answer Explanation

(1) A Directly covered both under the Constitution of India and the Indian
Citizenship Act.

(2) A Refer to Article 213 of the Constitution of India, 1950.

(3) A Refer to Article 342 of the Constitution of India, 1950.

(4) C Refer to Article 39A of the Constitution of India, 1950.

(5) A Refer to the Preamble of the Constitution of India, 1950. Inserted into
the Preamble through the 42nd Amendment Act.

36
(6) D It was set up on 6 August 1952 to strengthen and mobilize the effort
and resources of the nation in support of the Plan, to promote
common economic policies in all vital spheres, and to ensure the
balanced and rapid development of all parts of the country. It is
an extra-constitutional and non-statutory body. NDC is the listed as an
advisory body to Planning Commission but it's advice is not binding.

(7) C Refer to Article 75(1) of the Constitution of India, 1950.

(8) C Tamil was notified as a classical language in 2004 by the Government


of India. Odia was the latest language to be notified as a classical
language by the Government of India. As of now there are 6 classical
languages as notified by the Government of India.

(9) D There are 5 permanent members and all of them were essentially the
winners of World War – II who ratified the UN Charter and
established themselves as permanent members of the UN Security
Council.

(10) C The Champaran Movement was orchestrated between 1917-1918 as a


furtherance to the protest to the Indigo cultivation that was already
being done by the farmers in 1914.

(11) B The actual act was passed in 2005. Arvind Kejriwal was one of the
main leaders of the RTI Movement in India.

(12) A Raghavan was an Ex-Chief of CBI and placed his report in 2007. The
report was titled “The Menace of Ragging in the Educational
Institution and Measures to curb it”.

(13) D Very famously Shri LK Advani was appointed as the Deputy Prime
Minister although the Constitution of India does not mention any such
post.

(14) C Refer to Article 121 of the Constitution of India, 1950. The procedure

37
is largely the same as the impeachment for the President of India.

(15) A It is important to remember that other than these three countries we


have also borrowed from other countries like Ireland etc.

(16) A Repatriation is a process of returning a person to his /her place of


citizenship.

(17) C Article 110(3) of the Constitution says “If any question arises whether
a Bill is a Money Bill or not, the decision of the Speaker of the House
of the People thereon shall be final”.

(18) B Article 243-D of the Constitution mentions it; but 110th Constitutional
Amendment Bill is pending with the Parliament, where it is proposed
that one half of the seats shall be reserved for women in Panchayats.

(19) B Refer to Article 101 (4) of the Constitution of India, 1950.

(20) B Refer to Article 105 and Article 194 of the Constitution of India,
1950.

(21) B Zero Hour of the Parliament begins immediately after the Question
Hour and before the Agenda of the day. It starts at around 12 noon
(hence the name) and members can, with prior notice to the Speaker,
raise issues of importance during this time.

(22) C Refer to the case Hari Shankar Jain v. Sonia Gandhi.

(23) A Refer to Article 76 (1) of the Constitution of India, 1950.

(24) D Gupta Era was founded by Srigupta.

(25) C Objectives Resolution was moved by J.N Nehru on 13 December,


1946 and was unanimously adopted on 22nd January, 1947 by the

38
Constituent Assembly.
(26) C Refer to Article 32 and 226 of the Constitution of India, 1950.

(27) B Refer to Article 207(1) of the Constitution of India, 1950.

(28) C Refer to Article 251 of the Constitution of India, 1950.

(29) C Refer to Representation of People Act, 1951.

(30) D As per the Model Code of Conduct for guidance of political parties
and Candidates issued by Election Commission of India.

(31) C Compensation only for the inconvenience. Illness did not naturally
arose in the course of events.

(32) C The decision of the government is arbitrary and unreasonable by


placing a blanket ban.

(33) B Driver was not a party to the stealing.

(34) C The master could have foreseen the act of the stevedores.

(35) D Both Article 32 and 226 of the Constitution of India protect his
fundamental rights.

2013
Question Answer Explanation
number
71. C A is liable as he let the poisonous tree branch onto the premises of B,
and is consequently responsible for the death of cattle which fed on it.
72. D The work, which was delegated to B by A, was not performed in his
capacity as an agent or servant of the bank, rather performed in his

39
personal capacity.
73. A State would be held liable for the compensation, as it is vicariously
liable for the tortious act of the duty constable who wrongly
appropriated the goods of the plaintiff kept in the custody in
dereliction of duty
74. B Since the owners did not do anything wrongful in concert but their
dogs, the owners are not joint tortfeasors and therefore not liable
severally.
75. D The happening of such incident, and thereby the injury, was within the
foresight of defendant employees, and therefore, the act on the part of
municipal corporation was unreasonable
76. B B would be vicariously liable because there exists master-servant
relationship between B and the surgeon for the negligence of faulty
supply of oxygen on the part of the surgeon. The facts are silent on the
aspect of surgeon
77. A A is liable because he wrote the defamatory letter in Urdu knowing
that B did not know the Urdu language which amounts to constructive
publication
78. C D is liable for nuisance as he let the dust wrongfully escape and have
deleterious impact on P’s enjoyment of his premises.
79. B B is not liable for false imprisonment as the confinement of A in the
park was not without just cause or excuse.
80. B Railway Co. can’t be held liable for malicious prosecution, as it did
not have any malicious intent for getting B prosecuted. It merely had
an unverified and a mistaken knowledge and belief that the injuries
sustained by A were not real and were created by the doctor.
81. C A has committed the offence of attempt to murder as his act forms the
part of chronology of acts which would constitute actual commission
of the crime of murder if not interrupted as the poisoned water was
handed over to the bearer to serve B.
82. D Both X and Y are liable as procurement and administering of poison
to Z were done by X in furtherance of common intention of both X
and Y to kill Z.
83. A A is not guilty of theftas he did not take dishonestly the umbrella of Z
out of his possession without his consent. Rather, he misappropriated
the same when he did not restore it to Z when he got to know that it is
not his
84. C The daughter will not succeed as long as the reciprocal promise is
moved at her desire irrespective of the fact that it has not been

40
furnished by the beneficiary. However, the ans would A in strict
compliance with the principle.
85. D In the absence of contact on account of incompetence neither part has
any legal obligation.
86. B Article 23 of the Constitution of India provides prohibition on begar.
87. A Minority is a defense in case of criminal culpability. As per section
2(13) of Juvenile Justice (Care and Protection of Children) Act, 2015
it is determined on the date of commission of the crime.
88. B In GianKaurvs State of Punjab the Honorable Supreme Court held
that under Article 21 right to life does not include right to die. This
judgment overturned the decision of P Rathinamvs Union of India
wherein right to die was read into right to life.
89. B Eco mark is mark issued by Bureau of Indian Standards to products
conforming to a set of standards aimed at the least impact on the
ecosystem.
90. A Environment Impact Assessment or EIA can be defined as the study
to predict the effect of a proposed activity/project on the environment.
A decision making tool, EIA compares various alternatives for a
project and seeks to identify the one which represents the best
combination of economic and environmental costs and benefits.
91. D The Chipko movement or ChipkoAndolan was primarily a
forest conservation movement in India that began in 1973.
92. B As per the Explantion II to Section 2 of The Special Marriage Act-
1954 ‘Full blood’ and ‘half blood’- two persons are said to be related
to each other by full blood when they are descended from a common
ancestor by the same wife and by half blood when they are descended
from a common ancestor but by different wives
93. B A treaty is based on the consent of the parties to it, is binding, and
must be executed in good faith as per Latin Term pactasuntservanda
which means agreements must be kept as per Art 26 of Vienna
Convention of law of Treatise, 1969
94. A Sacrilege is covered under section 295A of IPC.
95. B A new chapter, that is Chapter XXII on Plea Bargaining has been
introduced in the Criminal Procedure Code. It was introduced through
the Criminal Law (Amendment) Act, 2005.
96. B See, Simpson & Group Companies Workers & Staff Union vs Amco
Batteries Ltd 1992 Karn.
When workers are dissuaded from reporting for work by stationing
certain men at the factory gates, such a step is known as picketing. If

41
picketing does not involve any violence, it is perfectly legal. It is
basically a method of drawing the attention of public towards the fact
there is a dispute between the management and the workers.
97. A Section 66F (B) of IT Act, 2000 defines cyber terrorism
98. B Obiter Dictum is the passing off remark of the judge, which is not
binding.
99. C Extradition may be briefly described as the surrender of an alleged or
convicted criminal by one State to another. In India the provisions of
Indian Extradition Act, 1962, govern the extradition of a fugitive from
India to a foreign country or vice-versa. The basis of extradition could
be a treaty between India and a foreign country.
100. D Present Chairperson of the 21st Law Commission is Former Supreme
Court judge Balbir Singh Chauhan.
101. C Refer to Article 300 under the Constitution of India for further details
on suits and proceedings by and against the state.
102. C Post dated Cheques have been read into Section 5 of Negotiable
Instruments Act, 1881 as per the holding in the case J Krishnan Nair
vs P Jasseentha and another by Kerala Court
103. D Intergenerational equity is a concept that says that humans hold the
natural and cultural environment of the Earth in common both with
other members of the present generation and with other generations,
past and future. It means that we inherit the Earth from previous
generations and have an obligation to pass it on in reasonable
condition to future generations.
104. D In 1980, in the Bachchan Singh case, the Supreme Court propounded
the “rarest of rare” doctrine and since then, life sentence is the rule
and the death sentence the exception.
105. A High Courts are constituted under article 214 of the Constitution of
India.

2014
Question Answer Explanation
number
71. D Akash will not succeed because he himself committed theft and is no
longer an invitee or visitor.
72. B Yes, Vijay’s claim will succeed because according to the principle an
easement is the right to use another person’s property for a stated
purpose.

42
73. B Yes the chemist will be held liable because he was aware of the side
effects of the ingredients and the consequences were reasonably
foreseeable.
74. B Yes it amounts to defamation because the guilt was not proved and
he was released by the court. Hence representation of the scene of
alleged murder displayed in Chamber of Horrors would amount to
defamation
75. A Both A and B would be liable because every partner is jointly as well
as severally liable.
76. C A is liable because supplying electricity is an inherently dangerous
use of land and he should have been careful.
77. A A is liable a B was his servant and committed the wrong in the
course of employment.
78. D Yes, the father is liable for the act of his child because he was
negligent in disposing off the gun.
79. A The doctor would be held responsible because the services rendered
by him falls under the definition of “services” under Consumer
Protection Act. Thus he is liable to pay compensation.
80. B Z is guilty for removing the kidney of A because doing this act for
money is an act not protected by principle 2.
81. C A is not guilty for the death of the child because he did not have the
intention and he did the act in good faith (Applying principle 1).
82. B No, A is not guilty because according to the principle he is bound by
law to do so.
83. B X is guilty of contempt of court because the accused was prosecuted
and he knew that the proceedings are pending before the court.
84. A A has not committed theft because he had no dishonest intention to
commit theft.
85. D Nandini will not succeed because preparation to commit an offence
is not an offence.
86. D Akash is not liable for using criminal force because he did not have
any malafide intention.
87. B The principle talks about giving the same amount of punishment and
penalty to a person as is applicable at the time of commission of the
offence. Therefore the boy should not be allowed the benefit of the
Probation of offenders Act.
88. B Aditya has been punished only once. There were two separate
proceedings, one is departmental and the other is court proceeding.

43
Departmental inquiry was an in house method by the company to
punish bad employees.
89. B A is not guilty of murder because the death of B was not a result of
his stabbing B but the negligence of hospital doctors.
90. C The contract has only restrained the parties from starting proceedings
in Udaipur. Pinto is free to begin proceedings in Jaipur. Thus this is
not a total restriction against approaching a tribunal for proceedings
and therefore the clause is not void.
91. A Inviting someone for dinner is just a social obligation and not a
legally enforceable contract.
92. C Dutt is not liable because he did not accept the offer given by Bakshi
but gave a counter offer which was not specific . Therefore there is
no contract in the present case.
93. D The fact that the initial agreement between Sunita & Neel was
against public policy makes the agreement void.
94. B The object being unlawful, the contract between Sunil &Anuj is
void.
95. C Every person is supposed to appear before a court of law when
summoned. Any consideration to do an act which a person is bound
to do by law is no consideration.
96. D Deep being a minor can neither enter into an agreement of agency
with Mandeep nor a general contract with any buyer.
97. A It is clear from the facts that Mani wanted to deal only with Pal.
Therefore Sam cannot accept the offer when it was not made to him
in the first place.Therefore Sam cannot recover the suit.
98. B Judicial standards accountability bill, 2010 aims to bring in standards
for judicial accountability. It does not give a definition of
misbehavior.
99. B Refer to case Novartis v. Union of India.
100. C Stalking and Voyeurism are non bailable offences i.e. a person
cannot ask to be released on bail as a matter of right.
101. A Uniform Civil Code has been provided under Art 44 of Indian
Constitution.
102. C NOTA is basically a voter’s right to reject.
103. B There are 11 fundamental duties provided under part IV A of Indian
Constitution.
104. C Sarkaria Commission was set up in June 1983 by the central

44
government of India. The Sarkaria Commission's charter was to
examine the relationship and balance of power between state and
central governments in the country and suggest changes within the
framework of Constitution of India.
105. D Planning commission is not a constitutional body. It was created by
the executive order of the Government of India.

2015
Question Answer Explanation
number
71. A The Principle is of Civil Defamation and as such does not require
intention, therefore publication is sufficient to constitute defamation.
72. B As per the Principle of Absolute Liabiility UCIL is liable to the
entire extent.
73. D In the facts it is given the partnership ends. After that the transactions
are independent and in personal capacity.
74. A (As per author the answer is incorrect as the pregnant woman’s
injury was not foreseeable, however stick to official key)
75. D Toleration would amount to invitation. Since he is aware that there
are people using the premises as a thoroughfare he should have taken
more care.
76. C The moment M boarded the bus he became a passanger and the
driver owes duty of care to all passangers.
77. A As per the principle legislation is a prerogative of the legislature and
not the court.
78. B Here the case of invitiation to offer as he just tells the price he is
willing to sell at, not that he agrees to sell .
79. C The burden shall be on X as he is usually sane so the law will also
presume him to be sane.
80. C A case of mistake of law and not fact, hence X will be liable
81. B Liable as the principle is of possession. Any intereference in
possession is sufficient to constitute trespass
82. D Clear application of the principles. Gunjan is a legitimate living son
and and the Rajat is only in the care of his sister, she is not the parent
or legal guardian.
83. B The intention is absent, and secondly he did not move it from
someone’s possession.

45
84. C As per principle upliftment and socially beneficient discrimination is
allowed but not on the basis of religion but on caste sex or class
85. A (The principle is not clear as to whether the deal by C was in
personal capacity or official capacity, if the deal was in official
capacity then the partnership firm would be liable and the correct
option would be B)
86. B Here Surender was to supply a particular rice which was no longer in
existence.
87. A Here Z was in the knowledge that the note belonged to some other
person.
88. D An option of sensitive plaintiff should have been given.
89. B
90. C B does not have the slave’s legal custody as slavery is not a legally
recognized relationship.
91. C Unlike the shopkeeper Roshini knew the note to be counterfeit
92. B Knowledge is intention. He knew that he was not allowed to carry
the knife
93. A Right to cause death by private defence is available in rape, sexual
offences, abduction or kidnapping.
94. B There was no necessity in this case as punching could be avoided.
95. A Yes. As the act had commenced the moment he planted the bomb.
96. C This bench comprised of J. Madan Lokur, & U.U. Lalit ans was to sit
every Friday at 2 p.m. but now it has been scrapped
97. C Self explanatory
98. B Self explanatory
99. A Self explanatory
100. A Self explanatory
101. C Self explanatory
102. B Self explanatory
103. A Self explanatory
104. B Self explanatory
105. B Self explanatory
106. A Self explanatory

46
2016
Question Answer Explanation
number
71. C The challenge is not sustainable because the policy of appointment of
only lady principal in women’s college is a reasonable classification
having a nexus with the object sought to be achieved under article 14
or 16 of Indian Constitution. Option b and c both are correct but
option c is closer to the principle.
72. D BCCI would not fall under the definition of “state” because
according to the principle the board is neither created under
Constitution or any statute neither it has been substantially financed
by the government. Thus the argument is not acceptable.
73. A Y can approach either Delhi Court or Mumbai Court because
according to the principle a suit can be instituted at two places.
Firstly, where the cause of action arises, secondly where the person
resides or carries business. In the given situation cause of action
arose in Mumbai and the defendant i.e. Z carries on business in
Mumbai. Hence both courts can be approached.
74. B J is not bound by the acceptance of K because according to the
principle acceptance is valid only if it is made in the way it is
expected to be i.e. via letter in the given situation. K gave the
acceptance through telephone and the letter did not reach within two
weeks hence J is not bound.
75. A The licensing power was granted by the Cinematograph Act. Any
withdrawal or transfer thereof was possible only through an
amending act and not by any rules made under the parent act.
76. D Rules are not violative of Art 15 because it does not provide
“Residence” as the ground of discrimination.
77. B The Company would be held liable because David committed the
wrong of giving the car to his brother during the course of
employment.
78. A According to the principle a Court can compel the witness only
within its local territory. Thus Puchu is not liable because he is not a
resident of Delhi.
79. D The law violates the constitutional provisions because it is laying
absolute restriction in the manufacturing of bidis. However the
object of getting more labours could be met merely by restraining the
employment during agricultural season only.
80. B X cannot claim the amount from KLM institute because the medical

47
institute instead of destroying the waste fibroids conducted research
and invented a new drug which was of no use for X.
81. D The statement is correct because the judgment determining
contractual obligations binds only the parties to a suit, hence is
judgment in personam whereas declaring a party to be insolvent is
judgment in rem.
82. D Yes the application can be allowed because retrospective application
of criminal law if it is beneficial to the accused is not against Article
20 clause 1 of the constitution.
83. B At times there can be incidental legislative encroachment by
legislative bodies on the matters enumerated under both state and
union lists
84. B A is Damnum Sine Injuria & R is Injuria Sine Damnum. Both are
independent legal propositions. A claim is only valid with Injuria
sine Damnum.
85. B Both the statements are independent of each other. Wagering
contracts are covered under sec 30 of Indian Contract Act.
86. B Both A & R are true but R is not the correct explanation of A
because both are essential conditions for valid customs however they
are independent of each other.
87. D Giving of thumb impression including other investigation
mechanisms like search and seizure are not violative of right against
self incrimination under Art 20 of the Indian Constitution
88. B Legislations under Indian law can be declared invalid only for
incompetency of legislature or violation of fundamental rights.
Incidental encroachments do not qualify as incompetency.
89. D There are various courts for resolution of International disputes
including but not limited to the International Court of Justice & the
International Criminal Court etc.
90. C The Constitution (119th Amendment) Bill has been passed by the
Parliament of India on 7th May 2015.While India will gain 510 acres
of land, ten thousand acres of land will notionally go to Bangladesh.
However, these are remote enclaves which India cannot access. This
legislation will redraw India’s boundary with Bangladesh by
exchanging enclaves in Assam, West Bengal, Tripura and
Meghalaya.
91. D Amitabh Kant is known to be the brain behind two key campaigns
of Narendra Modi government - Make in India and the ease of doing
business. Kant has been appointed in place of former IAS officer
Sindhushree Khullar, who was appointed as the first CEO of the

48
National Institution for Transforming India (Niti) Aayog, a body that
has replaced Planning Commission, for a fixed one-year term
beginning January 1, last year. Prime Minister Narendra Modi is
Chairman of Niti Aayog.
92. D High Court held that clause 2C-3 is unconstitutional and violated
fundamental rights. It has further observed that when married son
could get jobs on compassionate grounds then there is no reason to
deny the same to married daughter.
93. A Ruling conservative People’s Party (PP) of Spain led Prime Minister
Mariano Rajoy by has won the 2015 general election of the country.
In the general election, PP won the 123 seats by securing in the
lower house of the 350-member Parliament and securing 28.7
percent vote share.
94. A Pension fund regulatory body PFRDA has started using PAN instead
of Aadhaar for validation of new customers who can now be
registered online under the National Pension System (NPS) scheme.
95. B Hypothecation is the practice where (usually through a letter
of hypothecation) a debtor pledges collateral to secure a debt or as a
condition precedent to the debt, or a third party pledges collateral for
the debtor.
96. D The Securitisation and Reconstruction of Financial Assets and
Enforcement of Security Interest Act, 2002 (also known as
the Sarfaesi Act) is an Indian law .It allows banks and other financial
institution to auction residential or commercial properties to recover
loans.
97. A The Supreme Court on 8th October 2015 ordered to peruse the
National Green Tribunal (NGT) order directing the payment of
environmental compensation charge by commercial vehicles entering
Delhi, before proceeding with the hearing of a plea seeking levy of
similar charges on polluting vehicles.
98. D Apart from these some more legislations have been enacted like
ADHAAR Act etc.
99. D The states are Karnataka, Tamil Nadu and Odisha.
100. C Any of the two punishments can be granted.
101. B Minimum wages in Delhi are the highest in Northern India.
102. A GATT 1948, WTO 1995, UNCTAD 1964, NAFTA 1994.
103. C The Commercial Courts, Commercial Division and Commercial
Appellate Division of High Courts Bill, 2015 was introduced in
Rajya Sabha on April 29, 2015 by the Minister of Finance, Mr. Arun

49
Jaitley.
104. D Zero-rating (also called toll-free data or sponsored data) is the
practice of mobile network operators (MNO), mobile virtual network
operators (MVNO), and Internet Service Providers (ISP) not to
charge end customers for data used by specific applications or
internet services through their network, in limited or metered data
plans.
105. C Art 356 deals with the failure of the constitutional machinery of an
Indian state.

CLAT
2010
Question Answer Explanation
number
91 A Sajjadnashin/sadaqah is known as the manager of the
waqf board.
92 B It means by the very fact or the act itself
93 C Acquisition means taking control of the property and
ownership is permanent taking control of property
94 B Evidence that tends to support a proposition that is already
supported by some initial evidence, therefore confirming
the proposition.
95 D With respect to or in the interests of one side only or of an
interested outside party.
96 B Constitution of USA is federal in nature and Indian
Constitution is quasi-federal.
97 C Right to property is a legal right under Article 300-A.
98 A Marriage of Muslim male with any female of kitabia
religion, i.e., Christianity, Judaism, etc.
99 C Judicial Review was given by Marshall J. in the case
of Marsbury v. Madison.
100 B Ratio decidendi is binding in nature whereas an obiter
dictum is persuasive.
101 C Transfer of powers between Referred and Transferred
Subjects came to be known as Dyarchy or Dual
Government.
102 A Fringe benefit tax was imposed on 1st April, 2005.
103 B Transfer of ownership of property is a civil
proceeding.
104 C Shyam is the defendant.
105 D CrPC was implemented in the year 1973.
106 C Self explanatory
107 A Puisne judge is a judge without a distinction or title. This
was the title formerly used in English common law courts

50
for a judge other than a chief judge. Today, puisne
judge refers to any judge of the English High court, apart
from the chief justice.
108 A Ultra vires means outside the power.
109 B At the Time of the Commission of the Crime Only X
Committed the Crime Z only made an arrangement to
split the amount subsequently to the crime being
committed
110 B Moots in law schools are temporary legal problems
created in fictional courts.
111 B Scheduled Tribes status is always religious neutral.
112 A Anna Chandy- First female judge of HC
Fatima Beevi- First female judge of SC
113 C An item of property other than freehold land, including
tangible goods.
114 D Plaintiff-the person who files the suit
Defendant-the person against whom the suit is filed
115 C An accomplice to a felony who confesses his or her guilt
and gives evidence against his or her confederates.
116 D Refer to Article 65 of the Constitution of India, 1950.
117 D Self Explanatory
118 B Self Explanatory
119 A Child Marriage Restraint Act repealed Sharda Act,
1929.
120 C In this case the bag was sealed By opening the bag he
signified the intention to consume the entire goods as
the remainder of the contents could not be resold
121 B Self Explanatory
122 D ICJ is the principal judicial organ of United Nations.
123 B Rustom Cavasjee Cooper v. Union Of India (1970 AIR
564, 1970 SCR (3) 530), popularly known as the Bank
Nationalization case, held that the Constitution guarantees
the right to compensation, that is, the equivalent money of
the property compulsorily acquired.
124 D Self Explanatory
125 C Competition Act repealed MRTP Act.
126 A Self Explanatory
127 A Chief Information Commissioner is a statutory post
under RTI Act, 2005.
128 D RTI is a legal right.
129 B Relationship based on mutual trust and confidence.
130 D Self explanatory
131 C Absolute legal responsibility for an injury that can be
imposed on the wrongdoer without proof of carelessness
or fault.
132 D A mortgage or other claim on property or assets.
133 D Restitution of conjugal rights is filed under Sec 9 of
Hindu Marriage Act, 1955.
134 B In India, the grant of Parole is largely governed by the rules
made under the Prison Act, 1894.

51
135 C Acquital is when the person is acquitted of the
prosecution.
141 C The Manufacturer is the neighbor of the consumer, as
the manufacturer keeps the consumer in mind while
manufacturing the food. There is no direct contractual
relationship between them as it is the shopkeeper who
sells the good to the consumer
142 D Freedom connotes both the positive and negative
aspects as the principle requires a choice between both
143 B Here the payment of commission denoted a
relationship between the Bank and the agent However
be as that may option B is most close to the principle
in wording
144 C Everyone includes X
145 D He came with the consent which translates into an
invitation

2011

Question Answer Explanation


number
156 A There is a restriction but Principle D, the actions of private persons
are not covered principle
157 D Unlike the previous question here the obligation to join a union is
made by Parliament’s law, as such it would amount to an
administrative/state action
158 C A person also has the right with whom to associate If the trade union
does not have the right to choose it’s members then it would amount
to a case of violating the right of the trade union members to choose
whom to associate with
159 D Yet again the it is a private action not susceptible to fundamental
rights
160 C The Principle only protects the right to join and choose a trade union
The objective of trade union is to secure the rights of it’s members
through various mean like strike and therefore merely an objective
161 B Elizabeth has a better right as against the airline company
162 B As per the principal the property rights have no bearing upon an
unattended item
163 D There is nothing in the Principle which permits the right of the
airline company to exercise rights over unattended objects [Legally
speaking however the rights would vest with the airline company, but
we go strictly according to the principle]
164 A The release of all women prisoners is an order based on sex of the
principle and therefore directly discriminates on the basis of
Principle One
165 C It does discriminate on the basis of age, but it is not a ground
prohibited by Principle A, hence it cannot be described as

52
discriminatory
166 B Rule D requires an act to be absolutely necessary Release may or
may not be beneficial to women For example certain women how are
manic may be better off in a facility, hence B
167 B C seems to be a correct answer but isn’t as the statistic speaks of
women in Bihar, not in prison It may be the case that out of the
women in prison almost all the women may be graduate in which
case it will not be discriminatory but beneficial to them. However if
almost 90% persons being released are from upper castes then it is
indirectly discriminatory against the backward class
168 A Unlike question A the requirement of being absolutely necessary is
not present
169 D Under the Principle a person having a guardian is supposed to be
treated like a minor till s/he reaches the age of 21
170 D Bandita should have been the major and that she had the necessary
authority to sell the land
171 A As a major she would have the capacity to contract
172 C In the principle the contract is protected if the minor induced the
other person Here Ajay induced Bandita to enter into the contract
173 A The law is made to protect the interests of minors and permitting
Ajay would defeat the protection afforded to a minor
174 C A As per Principle A the person using the threat need not necessarily
be the person with whom the contract is entered with
175 C There is no existing relationship between Chulbul and Dhanraj
Aadil and Baalu are friends, not their fathers
176 D Being complicit implies both of them jointly coerced by Dhanraj
177 D He was complicit in the act of coercion by participation in the
abduction
178 B The effects must be uniform for all
179 D Everyone must have an equal opportunity to benefit
180 C Option A mandates the effect should benefit all
181 B Here an opportunity is given to all or not given at all and when
decided to be saved everyone is saved
182 D The quality test only accepts the product Different people will have
different ways of manufacturing a product but so long as it meets a
quality standard the product is acceptable as such there is no control
over manner and method over the product
183 D The definition of a workplace and the work hours would amount to
controls over the work style
184 C Having lunch outside the premises is not an incidental function
[Legally the position is difference if there is no canteen in the
premises, but as a CLAT student you are not expected to know that]
185 C As there existed no relationship between Ashish and the company
there could be no question of the action being incidental to rolling
biris or not
186 C The moment he boarded the company bus he would said to have
commenced employment
187 B He moved the painting but with the consent to move it
188 A As per the principal there must be an intent to damage or reduce the

53
value of the painting
189 C Here he would be guilty of theft Since Indira did not know about the
painting he would be removing it without her knowledge and consent
190 C Theft is a crime against possession The principle does not speak
about dishonest intent so it is irrelevant as to what intent Indira had
191 None Kamala’s actions would not constitute criminal damage by any
measure
192 D The carpet does not meet the requirement of a fixture,
193 B The door being affixed becomes a fixture a per Principle B
194 B Being placed or being attached makes a difference to the fixture
195 D As per principle C a good intended to be a part of the premises may
qualify as a fixture
196 C The Principle of ad infitum is given
197 D Hoisting a flag over 75 shall not constitute a necessary use
198 B In this case under Rule A she would be entitled use land infinitely
Under Rule B Javed and Sandeep cannot put the hoarding as it casts
a shadow and therefore violates the enjoyment of land held by Shazia
199 D Neither would constitute a reasonable use
200 D The shadow casts a permanent shadow on her terrace

2012
Question Answer Explanation
number
151. A As per the principle only the Parliament has the prerogative to make
the law on its own accord.
152. B The response only gave the price of the car, it was not accompanied
by willingness to sell.
153. B The contract with a minor is absolutely void.
154. A Clear application of the principle
155. D Article 15 and 16 which provide for reservations are not against
equality but in furtherance of equality.
156. A Protected by the Principle of necessity as given in the principle
157. C Rash is defined as acting or done without careful consideration of the
possible consequences; impetuous. As such his actions shall constitute
both a rash and a negligent act.
158. A The cutting of the tree is enough to satisfy the requirement that the
object of theft should move.
159. B Injuria sine damno. In either case the question is on violation of a
legal right not a fundamental right.
160. C In this case the baby was thrown not with the intention to kill him
Further he jumped himself which clearly indicates there was no

54
intention to kill.
161. C It was imperative upon X to become aware of the law before choosing
a flight which was to transit in India.
162. A Communication of the offer is not complete as knowledge of the
offeree is required against the principle.
163. C The part of the contract buying the horse would be valid. The
remainder part of the contract will be void
164. C Mere silence does not amount to fraud.
165. A An incorrect presumption is not the samething as legal justification.
Further removal from possession is sufficient to commit a tort.
166. B A void contract need not be void because it is illegal, it can be void on
account of various reasons. However an illegal contract will be void.
167. A 26th November is celeberated as law days as we gave ourselves to the
rule of the Constitution
168. A Self explanatory
169. B Both are independent characteristics of fundamental rights.
170. C An institution simply because of being a minority institution cannot
be denied aid.
171. A The right to issue remedied by means of writ is a central feature of the
Supreme Court as the guardian of the Constitution.
172. A
173. A The extent of harm cannot be predicted in torts therefore the damages
have to be unliquidated.
174. B Both are independent reasons for price rise.
175. A Under Section 98 a person’s right to self defence is not affected solely
because of the reason that the attacker did not have an intention to
commit a crime.
176. D Murder is a crime under Section 300 and 302, not Section 341.
177. A Concept of Quasi contract
178. B R does not explain A
179. A Imperative means a binding character. Austin had described law to be
a command of the sovereign where people have no option but to
warrant obedience to it.
180. A Where a group works in furtherance of a common intention the group
is also supposed to be held liable if the act is done as a part of a group

55
even out of sudden provocation
181. B
182. A By effect of Article 21 A.
183. A
184. A
185. C The Kailash Mansarovar Yatra is subsidised
186. C The second object insperable from the first.
187. B Communication to Raju should have been made by the Board.
188. C Damnum sine injuria. If he had suffered harm on account of a
stampede arising out of overcrowding a suit could have still lain.
However picking the pocket is not a direct consequence of crowding
189. C There was no intention.
190. A The best and only option which can be marked.
191. D
192. C Since one of the partners is insolvent the liability will be divided
between the other two
193. A Quasi contract
194. B They are two separate rights.
195. Y There was an attempt to move the stone.
196. C
197. A
198. A The doctors opined that an immediate operation was needed.
199. A
200. D

2013
Question Answer Explanation
number
151. D An offer must be accepted in the currency of the offer. When the
original offer was made the condition of payment by Shyam
amounted to a counter offer not an acceptance. Since the original
offer was replaced by a counter offer it came to an end and Shyam
cannot insist on the acceptance of an offer which has come to an end.

56
152. B An agreement for ten thousand is less than the offer but it is a validly
entered agreement and is binding upon the parties.
153. C The principle says “under all circumstances”.
154. C There seems to be an error in printing, and the ‘not’ in the fourth
option should be missing, If it were so then D would be the correct
option However given the options we have to choose as to what is
the strongest reason as to why Komal should not be given the watch,
the best answer seems to C.
155. D The notice is not an embargo to carry on the business. It only calls
upon the dyers and printers to carry out business in a manner which
respects the environment.
156. D In this case the contract for sale of the car is an independent contract
from the original debt. The principle requires the same contract
should confer a benefit upon Suresh. Between C and D, D is closer to
the principle.
157. D The principle states the English postal rule. The contract was
complete on the day of posting the letter and the place of posting the
letter.
158. C It was S’s workers who went on strike. P cannot be blamed for the
fault of S’s workers.
Note: In law there is usually a force majure clause, which states that
a contracting party is not responsible for things outside the control of
the contracting party. But since no such clause is mentioned in the
facts/principle C will be the correct answer.
159. C As per the principle the contract can be mad void even subsequently
when it becomes known to the parties. Here the end object of the
contract was converted to running a child labour operation from the
premises, hence he could request that the contract be made void.
160. C The principle imposes strict liability on Hari.
161. C The gift when made properly to R transfers the property completely
in the hands of R. Since R is now the owner he can choose to give or
not give his property to S.
162. C Clear application of the principle (The court does not uphold filmi
contracts.
163. A The principle requires the agreement must be compulsorily in
writing.
164. A Clear application of the principle. The facts themselves state that he
was of unsound mind when he entered the contract. Option B is also

57
correct but the principle does not speak of burden of proof.
165. C In the face of all incorrect options C seems to be best answer.
Though B sees to be a tempting option, the principle says the person
coercing cannot get the contract enforced. So it is not a question of
may, he cannot get the contract enforced at all.
166. D The principle does not speak of age but that of father son
relationship. In this case the facts clearly state the father misused his
position. A also seems to be a correct option that the question is not
whether the first contract is valid the question is as to whether the
second option is valid.
167. A Both A and C are correct options. However the question is whether
the act of X amounts to a valid consideration or not. Here X’s act
amounts to consideration when he abstained and therefore he can
recover the amount or not.
168. D More detailed between C and D.
169. A Since in the contract there is no intent to the contrary the
Government shall be the first owner.
170. B License grants an ‘interest’ whereas an assignment is of a more
larger nature. Here only a limited right to distribute in a smaller
territory is given therefore it would e more in the nature of a license
than an assignment.
171. C Textbook application of the principle of vicarious liability
172. C In the real legal world such a contract could not be used as a
trademark. But the facts clearly say that the description acquired a
secondary description.
173. D The principle is a little strange. Ordinarily theft would be complete
when the tree was severed and the case would be a case of theft.
However in the principle the tree only ‘becomes capabale of being
the subject of theft as soon as it severed from the earth’. Coupled
with the fact that the person has to ‘take away from the land of any
person’ a good capable of theft, theft s not be complete.
174. C There seems no alternate explanation as to how the abdominal pack
was left. Further the facts themselves declare the surgeon’s
complicity.
175. A Just stick to the principle. The principle says that the parent’s
consent is necessary. Since it was not present it would be a case of
kidnapping.
176. C The principle requires the intercourse should have been with a
woman or an animal, but in this case there were two men involved.

58
177. No
answer
178. D Clear application of the principle as A was a minor on the date of the
killing
179. C B’s consent is of no consequence as he was a minor
180. D The question is on private defence, not on insanity; therefore B will
not be an answer.
181. A Here silence amounts to speech as by remaining silent he conveys
the impression that there if nothing wrong with the horse.
182. B The action of the Professor was critical of the policies of the
Government and not the institution of the Government. Further there
was no intention to incite hatred
183. A The principle says ‘that complainant only’ signifying only one
complainant may be granted the license.
184. B His power did not extend to arrest but only seizure of goods as per
the principle.
185. C The only reason he could not take the jewel was because it was not
present; moreover the principle says “in attempt does any act towards
commission of the offence”. Here the box was broken with the intent
to access the goods of the box.
186. B As there was no intentention “intending to harm or knowing or
having reason to believe that such imputation will harm”. The
missing shoe only implied to a custom.
187. C Both A and B were employees of the same owner.
188. A Clear application of injuria sine damno. Facts /Principle is unclear as
to when nominal damages are due and when ordinary.
189. C B was entitled to end the contract as A himself had told him he was
not required
190. B Volenti non fit injuria
191. A The contract was impossible to perform as C was already dead
192. B The principle says that sunsequent ratification or approval is of no
avail
193. B B did not comply with the condition subsequent
194. A Relevant facts are those which can have an impact on the outcome
of the case: A permanent resident abroad who never visits
india=relevant because=how would wage war if he was not in India.
The charge/accusation against him is obviously valid and lastly his

59
taking part in an armed resurrection being the basis of the accusation
is relevant
195. A The onus is on the complainant as per the principle
196. C If there was no opportunity of physical relationship between X and
Y then Y is but obviously not carrying X’s child
197. B Based on earlier decided case the rationale is that there was no
illegality in carrying out business in a manner in times of scarcity
198. A Neither of them are minors and there is a threat to loose property if
either of them remarries such is a restraint against marriage and
therefore were invalid
199. A Clear application of principle. The situation was outside his control

200. A Only the Parliament has the prerogative to decide to make a law on
it’s own as per the principle

2014
Question Answer Explanation
number
151 C The principle requires that a proposal must be made by ‘one person’ to
‘another’ Since the hall was uninhabited and thus no proposal could
have been made
152 D The facts themselves indicate the setting to be a filial one rather than a
business arrangement Moreover the relationship is that of a son and
father which would indicate a social relationship. Lastly the setting, a
lazy breakfast, and the father making the utterance, ‘casually’ does not
indicate any intention to create a legally binding relationship
153 B As per the principle acceptance must be communicated to X or an
authorized agent of X. In this case however acceptance is
communicated to Z who is a stranger and therefore no valid contract
could arise between X and Y
154 B Here we have to interpret ‘reasonable time’ An acceptance made after
two years cannot be reasonable time by any standards especially if it is a
contract to simply sell equipment and that too the amount involved is
only Rs 1,000
155 C No valid contract could have arisen as A being a minor the contract
would have been ‘void’ from the very beginning
156 A As per the principle the adequacy of consideration is irrelevant, so long
as it is legal, valid and specified freely by the parties. In the facts given
there is nothing to indicate that ‘A’ did not give his consent freely, thus
the contract will be a valid one
157 C The object of the contract being to commit a crime and indemnity from
criminal consequences would make the object unlawful and as such the
contract will be void

60
158 D The principle though once again on consideration is slightly different
from the one given in the preceding question. In this question if the
object or consideration is opposed to public policy which implies those
acts which are unhealthy in public interest, a court can declare the
contract to be void. In this case the object of the contract is one which
promotes corruption.
159 D The contract is one where there is consent though it isn’t free.
Therefore the contract shall be ‘voidable’ at the option of Y, as Y gave
his consent under coercion
160 C Agreements in restraint of marriage are void, not voidable, hence D
cannot be a correct answer. Since the agreement puts a kind of restraint
on X , C is the correct option
161 A In this case the money was handed to Y, in Y’s personal capacity (as a
neighbour of X) and not as an employee of the bank. Therefore Y’s
actions cannot be said to be in the course of employment
162 B This is a common and foreseeable risk associated with cricket i.e. the
ball shall land in the stands while hitting a six, a four or unluckily
sometimes even in no balls. As a spectator R agreed to the risk
implicitly while consenting to watch the match in a stadium
163 B Clear application of the principle. It does not matter whether X was
aware of income tax law or not. He was bound to cough up his taxes
164 A The principle requires legal right to be violated in order for an injury to
be committed. In this case no doubt the setting up of the New Schools
affect the business of the established school and causes it financial
damage, but the principle is clear to the extent that damage without
injury does not result in an actionable claim
165 B In this case though no damage was caused to B (his preferred candidate
won the election), yet his legal right was violated as he had a legal right
to cast his vote. Therefore the violation of this right shall result in an
injury which entitles him to bring an action and recover
damages/compensation though he may have suffered no actual loss
166 A Even assuming if D was speaking the truth, the principle places a
burden of proving the truth on the defendant. The principle also makes
it abundantly clear that the defendant will be liable if s/he is unable to
prove his/her assertion. In this case D is unable to back his assertion as
clearly stated in the facts. In such a case he will liable to P
167 D The principle states that when a single gift is made partially of property
which the person making the gift owns at the time of making the gift
and partially of such property he shall acquire in the future only the first
part of such gift shall be valid. In this case X makes a gift to Y
consisting of a house which he owns and land which he is expected to
get. As per the principle only the gift of the house shall be a valid one
168 B As per the principle the burden of being judicious while selecting a
cloth for the purpose of uniforms lay with the buyer, which is A in this
case. Moreover B is not at fault in any case as B was neither informed
by A nor was there any other way in which he could know that A was
looking specifically for a cloth fit for sewing uniforms. It was A’s duty
to enquire about such fitness of the cloth
169 B Since ‘X’ never acquired a legal title himself, having stolen the goods
he could not pass a legal title to Y

61
170 A C and D are too vague to begin with and can be eliminated as options at
the very outset. Amongst A and B, read the principle and the question
very carefully. The manufactures makes any good keeping in mind the
fact that it is the consumer who ultimately purchases the good and
therefore must be conscious that any negligence on its account shall
directly affect the consumer. Therefore B cannot be a correct option,
which leaves us with A.
171 C Two distinct hints in the facts point to this answer. Firstly M was on a
lunch break, thus off duty and secondly there was no connection of
picking up a fight and carrying out D’s employment
172 D The last part of the principle clarifies that the right to live with human
dignity does not confer the right to die
173 A As per the principle trespass can be committed either by the person
himself or through tangible objects as well. By throwing stones
therefore A committed trespass
174 B What the principle is saying is that if one directly interferes with the
property of another the tort of trespass is committed. For example in Q
173, a person was pelting stones at someone’s property. However, if it
is not direct but a consequence of one’s actions, the act may amount to
nuisance. In this case the person does not plant the tree on someone
else’s property. The tree is planted on A’s land. But as a ‘consequence’
of A’s actions B’s enjoyment of his land is affected. Hence it would
amount to nuisance
175 B The principle is not on negligence and therefore we are not concerned
with the liability of the watchman. We are verifying whether the actions
of R amount to conversion. R’s actions were such that they led to
stealing of S’s bike. R may not have the intention nor may have
foreseen it, but nevertheless his act of leaving the bike outside the stand
land to it being stolen. Therefore R’s act amounts to conversion against
S.
176 B This is one of those rare questions which require a little insight (and
application of legal knowledge) in order to solve them. This tends to
happen for less serious crimes. Cognizable offences include waging war
against India, being member of an unlawful assembly, receipt of illegal
gratification by a public servant, murder etc Example of non cognizable
offences includes cheating, mischief, forgery etc In this case since the
crime is of a very serious nature, murder the police officer does not
require a warrant and may cause arrest on his own accord
177 C In this case all the three proceeded to Darshan’s house with a common
intention. Thus the first requirement of the principle is met. Thereafter
all of them proceeded to fire including Roshan thus becoming party to
the criminal act. Therefore as per the principle Roshan is equally guilty
irrespective of the fact whether his bullet hit the victim or not
178 B The communication was made in ‘good faith’
179 B Sawant had ‘wilfully misinformed’ the magistrate while being fully
aware of the fact that the act was not an accident but a murder
180 A K had the knowledge that he was suffering from Cholera at the time of
travelling. It is also common knowledge that while travelling in a train
he would be directly interacting with a large number of passengers and
that he would be using the common facilities thereby exposing his co-

62
passengers to a similar risk. Therefore K was negligent
181 C The facts themselves state that the driver was driving in a ‘rash and
negligent’ fashion’. Even if the same was not mentioned further
climbing on the footpath is rash and negligent by any standards (unless
being compelled to for some )
182 B In this case B did not know Z to be behind the bush, therefore B did not
shoot with the intention to kill. In fact it was A who was responsible for
the entire act as ‘ ‘A’, intending to cause, or knowing it to be likely to
cause Z’s death induces B to fire at the bush
183 B As per the principle theft is committed when the person committing the
theft ‘moves’ the property from the owners possession. In this case Z
himself gave the plate to A;therefore even if A’s actions may not be
justified in selling the plate elsewhere, he has not committed theft as
defined in the principle
184 C As the entire action was committed with a malafide intention of
misleading B by passing of Z’s property as his own
185 A As the palanquin was halted by A’s own bodily strength in order to rob
Z .As per the principle the employment of such force in order to commit
an offence constitutes criminal use of force
186 B In order to solemnize the marriage under the SMA,1954, the female
must be of 18 years of age which is not the case here
187 B Read the principle carefully Read the principle carefully. Desertion is a
withdrawal from the marriage itself and not from a place. Rohan was in
constant touch with his wife and wishes that she live with him, however
Rohan cannot be said that he has wilfully abandoned his wife
188 C As per the principle one of the conditions of taking a child is that the
child has not already been adopted. Therefore only the first adoption
shall be valid
189 A As per the principle the work created may not have a literary value. It is
sufficient that it is a fruit of hard labour or application of mind or a
product of method. Furthermore the examples of ectories have
themselves been given the principle
190 A The principle specifies “the exercise of criminal jurisdiction depends
upon the locality of the offence committed and not upon the nationality
or locality of the offender”. The offender was in Karachi but the victim
was cheated out of his money in Mumbai, therefore the cheating took
place in Mumbai and shall fall within the jurisdiction of IPC,1860
191 B All of them had the intention to commit the crime and all of them were
involved in planning. Even though they might have not committed the
crime but all of them shall be guilty of criminal conspiracy on account
of being involved.
192 C X did not have Y’s permission to take the watch
193 B The value of article does not matter. Since the sheet of paper was taken
without permission there shall be wrongful loss.
194 A As X’s unsoundness of mind does not restrict Y’s right of private
defence
195 C As per the principle the transferee in this case C may at his option
exercise the right even if A was to acquire an interest in filed titled Z
subsequently

63
196 B The deity does not qualify as a living person and as per the principle
transfer can only be made between one living person to another
197 C Since A’s transfer is not gratuitous it is not a gift. Since he does not
receive money in lieu it cannot be sale. The principle does not speak of
mortgage, therefore by elimination it can only be an exchange. Even by
selection of options, he has received shopping building in exchange for
his house thus exchange, or Option C will be the right answer
198 A The process of selection was different for other candidates as
contrasted to X as they had to face an extra examiner. Further as
examiner X was aware of the process and questions before hand and
also of the standard and performance of other candidates. Moreover the
board likely to be prejudiced, not withstanding his absence, while
awarding marks to another member of the selection board
199 C Since we are speaking of actions of the employer A cannot be an
option. Since there was no permanent closure D cannot be an option.
Since no person was fired B cannot be an option which only leaves us
with C.
200 A As suspension can be treated to be a temporary ‘discharge’ from work

2015
Ques. Answer Explanation
number
111 A Here the tree has been cut down thereby converting it into movable
property as such it will constitute theft
112 B The case of Ashby vs White where the right to vote a legal right is
breached without any damage As such it shall result in an actionable
claim
113 C The Contract can be entered into but since he is usually of sound
mind, the presumption will be that the contract was entered into
when he was of sound disposition. As such the contract can only be
avoided if he proves that the circumstances would be other way
round
114 B An invitiation to offer is one where a person solicits an offer Here
Shyamsunder only informs him the lowest price he is willing to
consider offers at, he does not agree to sell the land. Therefore it is
only an invitation to sell
115 A Since he was returning to an officially prescribed destination, he was
still in the course of employment and giving lift to the girlfriend shall
be an example of authorized act done wrongly
116 A Conflicting answers have been given for this question. It is best to
choose the latest official answer key. As such the underlying logic
could be that the onions seller must make sure that his customers do
not obstruct the other shopkeepers.
117 C Again the last year’s answer key had a different answer, this years
key had a different answer Any person who throws a child and then
jumps in the well for a bath cannot be presumed to be mentally
normal
118 C In this case it is a mistake of law. Even though it is difficult that

64
George could not educate himself as to the nature of law owing to the
short time of the law being in force it is still a citien’s responsibility
to be compliant of the law. A mistake of law being no excuse
Gerorge will be guilty
119 C As per the principle, the nervous shock induced by Krishnan could
not have been reasonably forseen by Lakshmi
120 B Here he kept the Halwa in the house and therefore did not do
anything more than preparation
121 B As to what Lucky means, only an opinion can be given
122 C It is a case of mere silence
123 A The principle is based on possession. Since the goods were in
possession of the garage owner, X is liable for trespass
124 D This is a case of sensitive plaintiff. Although C seems close to the
answer, it is not clear and therefore D is a better choice
125 B Both are different offences under different laws although the act
may be the same
126 D Custom only when recives legal sanction becomes law. For example
the necessity of solemnizing certain rituals in Hindus in certain
communities is necessary in order to recognize a marriage
127 C Idols are examples of juristic persons, persons who are not human
beings but still persons in the eyes of law Companies are another
example
128 B Assertion speaks of control not emancipation. Hence both are
correct but two independent purposes of the law
129 A Both statements are correct and related
130 A The moral culpability of the person remains the same
131 D The Constitution provides that a High Court must be there for every
state but is silent about it’s physical location. There are several
examples, for example the Bombay High Court is the HC for both
Maharashtra and Goa
132 D Th COM is responsible only to the Lok Sabha
133 D The political parties are at self discretion for allocation of seats.
However since memebership to the Union and State Legislature is
provided for in the Constitution a constitutional amendment is
imperative
134 A Both are correct and related
135 A The Republic day marks the birth of India as a republic which was a
result of the Constitution coming in force
136 A The preparation, namely running towards the well had started, but
she did not attempt jumping as she was stopped beforehand. Thus it
will not be a case of attempt or act
137 A The intention element was only to pick the pocket
138 A Here both intention and conduct is satisfied
139 A Legal Knowledge
140 C Legal Knowledge
141 A Legal Knowledge
142 B Legal Knowledge
143 The As such all three statements are correct
word is

65
compel
led
144 A BCI, it is the regulatory body laying down standards and norms for
rules of practice
145 A Legal knowledge
146 C In the list given the only person higher would be a Former President
147 C Legal knowledge
148 D An extradition treaty implies that criminals of X country when
taking refuge in Y countrycan be deported to X country if both of
them have an extradition treaty
149 C Literally implies debatable
150 B It refers to a convict If it were an accused the term would be bail
151 C Independent cause of actions. If Mr X was hurt due to a stampede
then he could have sued the railways.
152 3 All three are correct. See contract theory
153 A Legal knowledge
154 D All are correct
155 D All are correct Legal Knowledge
156 B The Chief Justice of a State. The CJI administers an oath to the
President
157 D It is an example of executive law making
158 B Legal Knowledge
159 A Any Union authority not only the Council of Ministers derive
authority from him
160 C Legal Knowledge

2016
Legal Answers Explanation
Section
questions
1. C As per the principle since A neither arrested the woman nor
reported about her to police he has committed the offence.
2. A Although the intense provocation was there but the sudden
element was missing as there was sufficient time lapse between
the provocative incident and the incident of killing.
3. D As per the given principle the conditions put on the contract can
be complied with after the happening of the event, which was
precisely done so and therefore, the contract comes into being.
4. D As per the principle sale of liquor is prohibited by law, and over
which no contract can happen and thereby no legal action for the
breach of the same.
5. C As per the principle since d intentionally pull the chair on which p
was about sit, and thereby made him fall and get hurt, his acts are
actionable in law.

66
6. C As per the principle since Himmesh has the sufficient
understanding about the nature and consequences of his conduct
as he immediately sold off the ring which he wrongly got hold of,
he can't claim immunity on the ground of him being below 12.
7. B This is not a case of assault as A's acts did not generate
apprehension in the mind of B of imminent application of force to
his body.
8. A In the given facts B has neither instigated A to kill C nor he
acquiesced to A's proposition to kill C.
9. D As per the given principle the communication of proposal is
completed when B read it.
10. B Since the shopkeeper did not object to the cheque handed over to
him and accepted that, it can be reasonably construed as his
acceptance by conduct for the same
11. C A never intended to physically hurt by throwing the papers at the
table, though it touched B. Therefore, A is not liable as his act his
very trivial in nature.
12. A B is bound to pay for the eatables because he derived benefit by
consuming them and those eatables were delivered to him non
gratuitously by A.
13. D A person can enter into a contract when he is in sound sate of
mind though generally he is in unsound state of mind.
14. C Since all the facts are alleged in existence and therefore, are
relevant.
15. B P's legal right to exercise the right to vote was violated, and
therefore, he has a cause of action.
16. A The territorial water is the territory of the country and any
consignment entering into it will be construed as import in India
as per the given principle.
17. B As per the second principle given ramesh will be held liable as he
asked the driver to indulge in careless act of driving fast
18. B A can't be held liable for theft as B is not the property or chattel of
his father C.
19. C This is a classic case of damnum sine injuria i.e. though he
suffered financially his no legal right is violated.
20. A In this case B gave his consent under the fear of death as a gun
was put on his head. Therefore, his consent was not freely
exercised.

67
21. C Promise to discover the treasure by magic is certainly impossible
to perform and therefore, is not enforceable.
22. C A will be held liable for defamation as he circulated pamphlets
wherein he conveyed imputation concerning A's reputation.
23. D Alia's acting is outside the ambit of copyright law because her
performance doesn't qualify work under the aforesaid law as per
the given principle.
24. C A has committed the offence of causing death as he starved his
daughter to death which is an omission on his part.
25. A M can’t be held liable for C’s act as the act was outside the course
of employment of C
26. A B’s act is not accidental as it is devoid of proper care and caution.
27. C B’s act amounts to trespass as he entered A’s bed room without
his permission
28. A A is not liable for defamation because he wrote the offensive
letter to B which does not amount to publication
29. A B’s act does not tantamount to acceptance because it is a counter
offer made by him as he did not accept the offer of rupees 500 for
a chair by A.
30. D XYZ Bank can legally claim the loaned money from Sabu
because his loan is collateral to a wagering agreement.
31. A The fancy alphabet B can be registered as a trademark as no
quality can be attributed to it.
32. D A can’t be held liable for fraud because his silence as to facts
likely to affect the decision of a person to enter into a contract
does not amount to fraud.
33. B Lady is entitled to claim compensation because she did not
assume the risk though she had the knowledge of the risk.
34. A A is guilty of misappropriation as on discovering his mistake,
without disclosing the actual facts, dishonestly sells the property
to a stranger.
35. A D could not be made liable for false imprisonment, as he did not
totally restrict P's movements
36. C Fiduciary relationship is a relationship based upon mutual trust
and confidence, for e.g. Lawyer and client, banker and its
customer, doctor and patient, etc.
37. C The doctrine of lis pendens has emerged out of the maxim “ut lite

68
pendent nihil inno weteur “ which means nothing new should be
introduced during the pendency of the suit over property.
38. B Under Article 32 and 226 of the Constitution one can directly
approach SC and HC respectively for constitutional remedies.
39. A The 99th constitutional amendment Act 2014 has been held
unconstitutional by SC in the case of Supreme Court Advocates
on Record Association and another vs union of India.
40. D As per Prohibition of Child Marriage Act, 2006 the marriage age
is fixed for a boy and a girl.
41. B Refer to Article 25 of the Constitution of India.
42. C Refer to Raghubhir Chaudhary vs State of Haryana.
43. B Freedom of speech and expression is guaranteed under Article 19
(1) (a) of the Constitution of India
44. B DPSPs are not justiciable under Article 37 of the Constitution of
India.
45. C It’s an essential part of the Principles of Natural Justice.
46. D Persona non grata means persons of no status.
47. B Quo Warranto is one of the five prominent constitutional writs
issued in case of infringement of constitutional rights.
48. A Obiter Dictum is the passing off remark of the judge, which is not
binding
49. D It is a rule of evidence.
50. B Refer to MC Mehta vs Union of India.

NUJS
2004
Q. no Answer Explanation
132. D Ashish Nandy is an Indian political psychologist, social theorist,
and critic. He is not associated with the world of law.
133. A General will of the people is a democratic approach to law and not
Marxist approach
134. D As per principles of Common Law.
135. D Refer to the Preamble of the Indian Constitution.
136. C Refer to Art 38 of Indian Constitution

69
137. D Fundamental duties were added by 42nd Amendment Act 1976.
138. A Cultural and Educational rights Preserve the right of any section
of citizens to conserve their culture, language or script,
and right of minorities to establish and administer
educational institutions of their choice.
139. B Refer to Art 165 of Indian Constitution.
140. None The Panchayati Raj Bill was adopted by 73rd Amendment Act
141. D Manu-smriti is the popular name of the work, which is officially
known as Manava-dharma-shastra. It is attributed to the legendary
first man and lawgiver, Manu.
142. C Rajashekhara was an eminent Sanskrit poet, dramatist and critic.
He was court poet of the Gurjara Pratiharas.
143. B Patriarchy is a social system in which males hold primary power,
predominate in roles of political leadership, moral
authority, social privilege and control of property.
144. D Refer to Art 38 of ICJ Statute
145. C Most favoured Nation clause is one of the three basic principle of
GATT
146. C Tokyo round reduced tariffs and established new regulations
aimed at controlling the proliferation of non-tariff barriers and
voluntary export restrictions. 102 countries took part in the round
147. B Human Rights Day commemorates the day on which, in 1948, the
United Nations General Assembly adopted the Universal
Declaration of Human Rights.
148. C Dr. S Radhakrishnan was the first chairman of Rajya Sabha
149. C The first state reorganization commission was first constituted
under the chairmanship of Chief Justice Fazal Ali
150. D Art 15 was amended and clause 5 was added
151. A Habeas corpus (Medieval Latin meaning literally "You may have
the body") is a recourse in law whereby a person can report an
unlawful detention or imprisonment before a court. It can be
issued by High court under Art 226 and Supreme court under Art
32
152. C Refer to Art 13 of Indian Constitution.
153. D Refer to Representation of People of Indian Act 1951.
154. C An electoral system in which parties gain seats in proportion to

70
the number of votes cast for them
155. D Civil servants while holding office of profit cannot become
members of parliament.
156. A Territorial waters or a territorial sea as defined by the 1982 United
Nations Convention on the Law of the Sea, is a belt of
coastal waters extending at most 12 nautical miles from
the baseline of a coastal state
157. B WHO stands for World Health Organisation
158. C The tribunal is an ad hoc court which is located in The Hague,
Netherlands. The Court was established by Resolution 827 of the
United Nations Security Council, which was passed on 25 May
1993.
159. B The Charter of the United Nations (also known as the UN
Charter) of 1945 is the foundational treaty of the United Nations,
an inter governmental organization
160. A The United Nations Security Council (UNSC) is one of the five
principal organs of the United Nations, charged with the
maintenance of international peace and security as well as
accepting new members to the United Nations and approving any
changes to its United Nations Charter.

2007
Q. No. Answer Explanation

(42) A Lex Loci is a Latin phrase which literally translates to “Law of the
Place”.

(43) B Please see the National Rural Employment Guarantee Act, 2005.

(44) A The first meeting was held on December 9, 1946 at the Central Hall of
the Parliament of India. Sachchidananda Sinha was elected temporary
president of the assembly.

(45) C Please see Article 32 and Article 226 of the Constitution of India, 1950.

(46) D The Kyoto Protocol is an international treaty which extends the 1992

71
United Nations Framework Convention on Climate Change (UNFCCC)
that commits State Parties to reduce greenhouse gases emissions, based
on the premise that (a) global warming exists; and (b) man-made
CO2 emissions have caused it.

(47) A Please see Section 3(2)(a) of The Protection of Human Rights Act, 1993.
The current Chairperson as on July 2016 is Justice H.L. Dattu.

(48) D The NHRC is not a commission to review the working of the


Constitution.

(49) C The 42nd Constitutional Amendment also added the word “Socialist” to
the Preamble along with the word “Secular”.

(50) A Please see Article 124(2) of the Constitution of India, 1950.

(51) B Please see Article 33 of the Agreement on Trade-Related Aspects of


Intellectual Property Rights (the “TRIPS”), which is Annexure 1C of the
Agreement Establishing The World Trade Organisation, 1995.

(52) B Please see Article 145(3) of the Constitution of India, 1950.

(53) C The case of Holiness Kesavananda Bharati Sripadagalvaru &


Ors. vs. State of Kerala & Anr, (1973) 4 SCC 225 was decided on April
24, 1973 by a 13 judge bench of the Supreme Court.

(54) B Please see Section 2(h) of the Indian Contract Act, 1872.

(55) C Inserted by Section 2 of the Constitution (Eighty-sixth Amendment) Act,


2002.

(56) C Article 65 of Indian Constitution confers power on Vice President to act


as President in case later is absent. Similarly, Article 70 of the
Constitution confers power on Parliament to make provisions when even
Vice President is not available. Accordingly, The President (Discharge

72
of Functions) Act, 1969 of India provides that the Chief Justice of India
(CJI) (or senior most judge of SC in absence of CJI ) shall act as the
President of India in the event of the offices of both the President and
the Vice President being vacant.

(57) A Please see Article 109 of the Constitution of India, 1950.

(58) D Effective control of the Armed Forces, as a consequence of the various


Constitutional provisions including but not limited to Article 74, is with
the Council of Ministers headed by the Prime Minister.

(59) A Please see Article 19(1)(a) of the Constitution of India, 1950.

(60) B Please see Section 124A of the Indian Penal Code, 1860.

(61) B The party filing the appeal is called an Appellant and as a consequence
the other party is referred to as the Respondent.

(62) B These would be acts which are against public policy and hence would be
treated as void agreements under Section 23 of the Indian Contract Act,
1872.

(63) C The book was originally published in 1971 and revised in both 1975 (for
the translated editions) and in 1999.

(64) B Please see Article 165 of the Constitution of India, 1950.

(65) D Please see Article 38 of the Statute of the International Court of Justice.

(66) C The Supreme Court of India has the power to issue writs in the nature
of habeas corpus, mandamus, prohibition, quo warranto and
certiorari under article 32 of the Indian Constitution. Furthermore every
High Court of India has the power to issue writs in the nature

73
of habeas corpus, mandamus, prohibition, quo warranto and
certiorari under article 226 of the Indian Constitution.

(67) A Please see Section 2 of The Muslim Personal Law (Shariat) Application
Act, 1937.
(68) B The burden of proof under Indian Criminal Law is on the prosecution.

(69) B Tort law is made suitable to the Indian conditions appeasing to the
principles of justice, equity and good conscience and as amended by the
Acts of the legislature. Its origin is linked with the establishment of
British courts in India.

(70) B Ronald Dworkin has written a masterful explanation of how the Anglo-
American legal system works and on what principles it is grounded.
(71) C The Universal Declaration of Human Rights (UDHR) is a declaration
adopted by the United Nations General Assembly on 10 December 1948
at the Palais de Chaillot, Paris. The Declaration consists of thirty articles
which have been elaborated in subsequent international treaties,
economic transfers, regional human rights instruments, national
constitutions, and other laws.

(72) C It was introduced by the Nehru Government, on 10 May 1951. In a


landmark ruling on 11 Jan 2007, The Supreme Court ruled that ‘If law
put in 9th Schedule abridge or abrogate fundamental rights resulting in
violation of the basic structure of the Constitution, such laws need to be
invalidated. ’
(73) C It speaks about the power of Magistrates to issue order in urgent cases of
nuisance of apprehended danger in order to maintain public order.

(74) D In India ‘ex post facto law’ has been adopted in the Article 20 (1) which
states that No person shall be convicted of any offence except for
violation of a law in force at the time of the commission of the act
charged as an offence, nor be subjected to a penalty greater than that
which have been inflicted under the law in force at the time of
commission of the offence.

(75) A Hindu law (Hindu law Acts 1955-56), Muslim law (Muslim personal

74
laws (Shariat) Application Act, 1937), Christian law, Parsi law (Parsi
Marriage and Divorce Act, 1936).

(76) D National emergency in India is declared under Article 352 of the


Constitution. Further, as a result of Article 359 of the Constitution, all
Fundamental rights except Article 20 and 21 can be suspended during a
National Emergency. Such an emergencies were declared in India in
1962 (Indo-China war), 1971 (Indo-Pakistan war), and 1975 (Indira
Gandhi Emergency).

75

You might also like